P1 2011 Test 5 Answ
P1 2011 Test 5 Answ
P1 2011 Test 5 Answ
Questionnaire
Eléments de restitution
Restitution
P1 - 1 -- Quel type d'activités peuvent être effectuées dans le cadre de l'audit P1 - 1 -- Internal audit activities may involve which of the following?
interne ?
A. Des activités d'assurance. A. Assurance services.
B. Des activités de conseil. B. Consulting services.
C. Des activités d'assurance et de conseil. C. Both assurance and consulting services.
D. Ni des activités d'assurance, ni des activités de conseil. D. Neither assurance nor consulting services.
La réponse (A) est fausse. L'audit interne peut effectuer aussi bien des Answer (A) is incorrect because internal audit activities may involve
activités d'assurance que de conseil. both assurance and consulting activities.
La réponse (B) est fausse. L'audit interne peut effectuer aussi bien des Answer (B) is incorrect because internal audit activities may involve
activités d'assurance que de conseil. both assurance and consulting activities.
La réponse (C) est juste. L'audit interne peut effectuer aussi bien des Answer (C) is correct. Internal audit activities may involve
activités d'assurance que de conseil. Les activités d'assurance sont un both assurance and consulting services. Assurance services
examen objectif d'éléments probants, effectué en vue de fournir à are objective examinations of evidence for the purpose of
l'organisation une évaluation indépendante des processus de providing an independent assessment on risk management,
management des risques, de contrôle ou de gouvernement control, or governance processes for the organization.
d'entreprise. Les activités de conseil sont les conseils et services y Consulting services are advisory and related client service
afférents rendus au client donneur d'ordre, dont la nature et le champ activities, the nature and scope of which are agreed upon with
sont convenus au préalable avec lui. Ces activités ont pour objectifs de the client and are intended to add value and improve an
créer de la valeur ajoutée et d'améliorer le fonctionnement de organization's operations.
l'organisation. Answer (D) is incorrect because internal audit activities may involve
La réponse (D) est fausse. L'audit interne peut effectuer aussi bien des both assurance and consulting activities.
activités d'assurance que de conseil.
P1 - 2 -- Laquelle des propositions suivantes décrit le mieux l'objectif de l'audit P1 - 2 -- Which of the following best describes the purpose of the
interne ? internal audit activity?
A. Créer de la valeur ajoutée et améliorer les opérations d'une organisation. A. To add value and improve an organization's operations.
B. Aider la direction dans la conception et la mise en oeuvre des systèmes de B. To assist management with the design and implementation of
gestion des risques et de contrôle. risk management and control systems.
C. Examiner et évaluer le système comptable d'une organisation, à titre de C. To examine and evaluate an organization's accounting system
service rendu à la direction. as a service to management.
D. Superviser le système de contrôle interne d'une organisation pour le D. To monitor the organization's internal control system for the
compte d'auditeurs externes. external auditors.
La réponse (A) est juste. D'après sa définition, l'audit interne est “une Answer (A) is correct. According to the definition of internal
activité indépendante et objective qui donne à une organisation une auditing, "Internal auditing is an independent, objective
assurance sur le degré de maîtrise de ses opérations, lui apporte des assurance and consulting activity designed to add value and
conseils pour les améliorer et contribue à créer de la valeur. L'audit improve an organization's operations. It helps an organization
interne aide cette organisation à atteindre ses objectifs en évaluant, par accomplish its objectives by bringing a systematic,
une approche systématique et méthodique, ses processus de disciplined approach to evaluate and improve the
management des risques, de contrôle et de gouvernement d'entreprise, effectiveness of risk management, control, and governance
et en faisant des propositions pour renforcer leur efficacité.” processes."
La réponse (B) est fausse. Ces fonctions de conception et de mise en oeuvre Answer (B) is incorrect because performing the functions of design
des systèmes de gestion des risques et de contrôle entraveraient l'objectivité and implementation of risk management and control systems
des auditeurs internes. Un auditeur interne peut recommander des normes would impair the objectivity of the internal auditors. An internal
de contrôle et examiner les procédures avant leur mise en oeuvre. auditor may recommend control standards and review procedures
La réponse (C) est fausse. L'audit interne est bien plus vaste que les simples prior to their implementation.
missions d'examen et d'évaluation d'un système comptable. Answer (C) is incorrect because internal auditing is much broader
La réponse (D) est fausse. L'audit interne sert l'organisation, et non les than examining and evaluating an organization's accounting
auditeurs externes. system.
Answer (D) is incorrect because internal auditing serves the
organization, not the external auditors.
Restitution
P1 - 4 -- Lequel des postes suivants est responsable du développement et du P1 - 4 -- Which of the following is responsible for developing and
maintien d’un programme d’assurance et d’amélioration de la qualité qui maintaining a quality assurance and improvement program that
couvre tous les aspects de l’audit interne et qui permet un suivi permanent de covers all aspects of the internal audit activity and continuously
son efficacité ? monitors its effectiveness?
A. La direction générale. A. Senior management.
B. Le responsable de l’audit interne. B. Chief audit executive.
C. Le conseil d’administration. C. The Board of Directors.
D. Le comité d’audit. D. Audit committee.
La réponse (A) est fausse. Ils ne sont pas responsables du développement et Answer (A) is incorrect because they are not responsible for
du maintien d’un programme d’assurance et d’amélioration de la qualité qui developing and maintaining a quality assurance and improvement
couvre tous les aspects de l’activité de l’audit interne et qui permet un suivi program that covers all aspects of the internal audit activity and
permanent de son efficacité. continuously monitors its effectiveness.
La réponse (B) est juste. Le responsable de l’audit interne doit élaborer et Answer (B) is correct. The chief audit executive should develop
tenir à jour un programme d’assurance et d’amélioration qualité portant and maintain a quality assurance and improvement program
sur tous les aspects de l’audit interne et permettant un contrôle continu that covers all aspects of the internal audit activity and
de son efficacité. L’objectif d’un tel programme est d’aider l’audit interne à continuously monitors its effectiveness. The program should
apporter une valeur ajoutée à améliorer le fonctionnement de be designed to help the internal auditing activity add value and
l’organisation et de garantir que l’audit interne est en conformité avec les improve the organization's operation and to provide assurance
Normes et Le Code de Déontologie. that the internal audit activity is in conformity with the
La réponse (C) est fausse. Ils ne sont pas responsables du développement Standards and the Code of Ethics.
et du maintien d’un programme d’assurance et d’amélioration de la qualité Answer (C) is incorrect because they are not responsible for
qui couvre tous les aspects de l’activité de l’audit interne et qui permet un developing and maintaining a quality assurance and improvement
suivi permanent de son efficacité. program that covers all aspects of the internal audit activity and
La réponse (D) est fausse. Ils ne sont pas responsables du développement continuously monitors its effectiveness.
et du maintien d’un programme d’assurance et d’amélioration de la qualité Answer (D) is incorrect because they are not responsible for
qui couvre tous les aspects de l’activité de l’audit interne et qui permet un developing and maintaining a quality assurance and improvement
suivi permanent de son efficacité. program that covers all aspects of the internal audit activity and
continuously monitors its effectiveness.
Restitution
P1 - 6 -- Selon le Cadre de Référence International des Pratiques P1 - 6 -- According to the International Professional Practices
Professionnelles, l'indépendance de l'audit interne passe par : Framework, the independence of the internal audit activity is
achieved through:
La réponse (A) est fausse. La dotation en personnel et la supervision sont Answer (A) is incorrect. Staffing and supervision relate to the
liées à la compétence professionnelle de l'audit interne. professional proficiency of the internal audit activity.
La réponse (B) est fausse. la formation professionnelle continue et la Answer (B) is incorrect. Continuing professional development and
conscience professionnelle sont liées à la compétence professionnelle de due professional care relate to the professional proficiency of the
l'audit interne. internal auditor.
La réponse (C) est fausse. Les relations humaines et les communications Answer (C) is incorrect. Human relations and communications
sont liées à la compétence professionnelle de l'audit interne. relate to the professional proficiency of the internal auditor.
La réponse (D) est juste. Selon le CRIPP, la position dans l'organisation Answer (D) is correct. According to IPPF, organizational
et l'objectivité permettent aux auditeurs internes de porter des status and objectivity permit members of the internal audit
jugements sans partialité et sans préjugé, ce qui est essentiel à la activity to render the impartial and unbiased judgments
bonne conduite des audits. essential to the proper conduct of engagements.
Restitution
P1 - 8 -- Dans laquelle des situations suivantes l'auditeur risque-t-il de P1 - 8 -- In which of the following situations would an auditor
manquer d'objectivité ? potentially lack objectivity?
A. Un auditeur examine les procédures relatives à une nouvelle connexion A. An auditor reviews the procedures for a new electronic data
pour l'échange de données électronique avec un client important avant sa interchange connection to a major customer before it is
mise en service. implemented.
B. Quatre mois après avoir été transféré dans le service de l'audit interne, un B. A former purchasing assistant performs a review of internal
ancien responsable des achats analyse les contrôles internes appliqués aux controls over purchasing four months after being transferred to the
achats. internal audit activity.
C. Un auditeur recommande la mise en place de normes de contrôle et de C. An auditor recommends standards of control and performance
mesures de la performance pour un contrat avec une société de services measures for a contract with a service organization for the
concernant le traitement des fiches de paie et des avantages sociaux. processing of payroll and employee benefits.
D. Un membre du service de la paie aide un auditeur à vérifier le stock D. A payroll accounting employee assists an auditor in verifying the
physique de petits moteurs. physical inventory of small motors.
La réponse (A) est fausse. Selon le CRIPP, “l'objectivité de l'auditeur interne Answer (A) is incorrect. The IPPF states that the internal auditor's
n'est pas compromise lorsqu'il est amené à examiner des procédures avant objectivity is not adversely affected when the auditor reviews
leur mise en application.” procedures before they are implemented.
La réponse (B) est juste. Selon le CRIPP, “les membres du personnel Answer (B) is correct. The IPPF states that persons
qui sont affectés au service d'audit interne ne doivent pas participer transferred to the internal audit activity should not be
avant un délai raisonnable (au moins un an) à l'audit de leurs assigned to audit those activities that they previously
précédentes activités.” performed until a reasonable period of time (at least one year)
La réponse (C) est fausse. Selon le CRIPP, “l'objectivité de l'auditeur interne has elapsed.
n'est pas compromise lorsqu'il est amené à recommander la mise en place Answer (C) is incorrect. The IPPF states that the internal auditor's
de normes de contrôle pour les systèmes d'information avant leur mise en objectivity is not adversely affected when the auditor recommends
application.” standards of control for systems before they are implemented.
La réponse (D) est fausse. Faire appel à des membres d'autres services pour Answer (D) is incorrect. Use of staff from other areas to assist the
aider l'auditeur interne ne compromet pas l'objectivité de ce dernier, en internal auditor does not impair objectivity, especially when the staff
particulier si ces personnes n'émanent pas du service audité. is from outside the area being audited.
Restitution
P1 - 10 -- Un auditeur interne chargé de vérifier qu'un fournisseur respecte P1 - 10 -- An internal auditor assigned to audit a vendor's
bien les normes de qualité des produits se trouve être le frère du contrôleur compliance with product quality standards is the brother of the
de gestion de ce fournisseur. Quelle attitude l'auditeur interne doit-il adopter ? vendor's controller. The auditor should:
A. Accepter la mission, mais éviter les contacts avec le contrôleur de gestion A. Accept the assignment, but avoid contact with the controller
pendant le travail sur le terrain. during fieldwork.
B. Accepter la mission, mais faire part de son lien de parenté dans les B. Accept the assignment, but disclose the relationship in the
communications finales de la mission. engagement final communication.
C. Avertir le fournisseur qu'il y a un risque de conflit d'intérêt. C. Notify the vendor of the potential conflict of interest.
D. Avertir le responsable de l'audit interne qu'il y a un risque de conflit D. Notify the chief audit executive of the potential conflict of interest.
d'intérêt.
La réponse (A) est fausse. Même si l'auditeur évite les contacts avec le Answer (A) is incorrect. Even if the auditor avoided contact with the
contrôleur de gestion, on aura tout de même l'impression qu'il y a un conflit controller, there would still be the appearance of conflict of interest.
d'intérêt. Answer (B) is incorrect. Situations of potential conflict of interest or
La réponse (B) est fausse. Les situations de conflits d'intérêt ou de préjugés bias should be avoided, not merely disclosed.
potentiels doivent être évitées, et non simplement mentionnées. Answer (C) is incorrect. Conflicts of interest should be reported to
La réponse (C) est fausse. Les conflits d'intérêt doivent être signalés au the chief audit executive, not the vendor or engagement client.
responsable de l'audit interne, et non au fournisseur ou au client de la Answer (D) is correct. The IPPF states that internal auditors
mission. should report to the chief audit executive any situations in
La réponse (D) est juste. Selon le CRIPP, les auditeurs internes doivent which a conflict of interest or bias is present or may
signaler au responsable de l'audit interne toutes les situations où l'on reasonably be inferred.
peut raisonnablement suspecter l'existence d'un conflit d'intérêt ou
d'un parti-pris.
Restitution
A. Transmettre immédiatement cette communication à l'Institut de l'Audit A. Immediately report the communication to The Institute of Internal
Interne, lui en demander une interprétation déontologique et l'interroger sur la Auditors and ask for an ethical interpretation and guidance.
marche à suivre. B. Inform the president that this scope limitation will need to be
B. Informer le président que cette limitation du champ d'intervention devra reported to the board.
être notifiée au Conseil. C. Continue to investigate the area until all the facts are determined
C. Continuer d'enquêter sur cette affaire jusqu'à ce que les faits soient and document all the relevant facts in the engagement records.
confirmés ou infirmés et consigner tous les éléments pertinents dans le D. Immediately notify the external auditors of the problem to avoid
dossier de la mission. aiding and abetting a potential crime by the organization.
D. Informer immédiatement les auditeurs externes du problème, afin d'éviter
d'encourager un délit potentiel de l'organisation et de s'en rendre complice.
La réponse (A) est fausse. L'IIA n'a pas autorité en la matière. Answer (A) is incorrect because The IIA has no authority in this
La réponse (B) est juste. Une limitation du champ d'intervention et ses matter.
répercussions possibles doivent être communiquées, de préférence Answer (B) is correct. A scope limitation along with its
par écrit, au Conseil, au comité d'audit ou à tout autre organe délibérant potential effect should be communicated, preferably in
approprié (CRIPP). writing, to the board, audit committee, or other appropriate
La réponse (C) est fausse. Le responsable de l'audit interne doit au préalable governing authority (IPPF).
consulter le conseil. Il crée de la valeur ajoutée en servant l'organisation et le Answer (C) is incorrect because the CAE should first consult the
conseil peut, en fait, être pleinement conscient du problème mais ne pas board. The CAE adds value by serving the organization, and the
vouloir encourir de frais supplémentaires. board may, in fact, be fully aware of the problem and may not want
La réponse (D) est fausse. Le travail est préliminaire, et l'auditeur interne n'a to incur additional costs.
pas encore étayé son opinion. Il serait donc prématuré de contacter les Answer (D) is incorrect because the engagement work is preliminary,
auditeurs externes. Néanmoins, si ces derniers procèdent à une enquête, les and the internal auditor has not yet formed a basis for an opinion.
auditeurs internes doivent leur communiquer les travaux réalisés à ce jour. Thus, contacting the external auditors is premature. However, if an
inquiry is made by the external auditors, the internal auditors should
share the work done to date.
Restitution
P1 - 13 -- La place de l'audit interne dans l'organisation : P1 - 13 -- The organizational status of the internal audit activity
A. Doit être suffisante pour lui permettre de remplir ses fonctions. A. Should be sufficient to permit the accomplishment of its
B. Est optimale lorsqu'il rend directement compte au conseil d'administration. responsibilities.
C. Nécessite, annuellement, l'approbation des programmes de travail de B. Is best when the reporting relationship is direct to the board of
l'audit interne, des prévisions d'effectifs et du budget financier par le conseil. directors.
D. Est garantie lorsque la charte définit spécifiquement son indépendance. C. Requires the board's annual approval of the engagement work
schedule, staffing plan, and financial budget.
D. Is guaranteed when the charter specifically defines its
independence.
La réponse (A) est juste. “L'audit interne doit être indépendant” Answer (A) is correct. The IAA should be independent (IPPF).
(CRIPP). “Les auditeurs internes sont indépendants lorsqu'ils peuvent Internal auditors are independent when they can carry out
exercer leur activité librement et de façon objective. Cet objectif est their work freely and objectively, and independence is
atteint grâce à leur position dans l'organisation et à leur objectivité.” achieved through organizational status and objectivity (IPPF).
(CRIPP). Ainsi, “le responsable de l'audit interne doit relever d'un Thus, the chief audit executive should report to a level within
niveau hiérarchique permettant aux auditeurs internes d'exercer leurs the organization that allows the IAA to accomplish its
responsabilités” (CRIPP). “Les auditeurs internes doivent bénéficier du responsibilities (IPPF). Internal auditors should have the
soutien de la direction générale et du Conseil afin d'obtenir la support of management and of the board so that they can
coopération des audités et de pouvoir exercer leur activité sans gain the cooperation of engagement clients and perform their
entrave” (CRIPP). work free from interference (IPPF).
La réponse (B) est fausse. L'audit interne doit bénéficier d'un soutien Answer (B) is incorrect because the IAA requires day-to-day
quotidien qui ne peut lui être fourni par le conseil. Il doit tout de même rendre support that cannot be provided by the board. It should still report to
compte à la direction. Dans l'idéal, l'audit interne doit être rattaché management. Ideally, the IAA should report administratively to the
administrativement au directeur général de l'organisation. CEO of the organization.
La réponse (C) est fausse, car “le responsable de l'audit interne doit Answer (C) is incorrect because the CAE should submit annually to
soumettre, annuellement, à la direction générale pour approbation et au senior management for approval, and to the board for its
Conseil pour information, les programmes de travail de l'audit interne, les information, a summary of the IAA's work schedule, staffing plan,
prévisions d'effectifs et le budget financier” (CRIPP). and financial budget (IPPF).
La réponse (D) est fausse. Un fait énoncé dans la charte ne garantit pas Answer (D) is incorrect because a statement in the charter does
l'indépendance. not guarantee independence.
Restitution
La réponse (A) est fausse. La limitation devra être communiquée en premier Answer (A) is incorrect because the limitation should be
lieu au conseil d’administration. communicated first to the board.
La réponse (B) est fausse. Il n’y a aucune obligation ni aucun besoin de Answer (B) is incorrect because there is no requirement or need to
communiquer la limitation à l’auditeur externe. communicate the limitation to the external auditor.
La réponse (C) est juste. Les plannings des travaux de mission les Answer (C) is correct. Engagement work schedules, staffing
plannings des effectifs ainsi que les budgets devraient faire état vis-à- plans, and financial budgets should inform senior
vis de la direction générale et du conseil d’administration de l’étendue management and the board of the scope of internal auditing
du travail de l’audit interne et de toute limitation imposée sur ce champ work and of any limitations placed on that scope (IPPF).
d’action (CRIPP). De plus une limitation à un champ d’action avec son Furthermore, a scope limitation along with its potential effect
impact éventuel devra être signalé de préférence par écrit au conseil should be communicated, preferably in writing, to the board,
d’administration au comité d’audit ou à toute autre organe délibérant audit committee, or other appropriate governing authority
(CRIPP). (IPPF).
La réponse (D) est fausse. L’AI existe afin d’aider l’organisation à atteindre Answer (D) is incorrect because the IAA exists to help the
ses objectifs. Les auditeurs internes devront donc communiquer ses conflits organization achieve its objectives. Thus, the internal auditors
avec la direction au conseil d’administration. should communicate with the board about conflicts with
management.
P1 - 15 -- À partir du moment où le Responsable de l’Audit Interne reçoit P1 - 15 -- After the Chief Audit Executive gets approval from the
l’approbation du Conseil d’Administration pour fournir des services de conseil Board of Directors to offer consulting services, what should be
quelle action doit être entreprise ? done?
A. Le RAI devrait commencer à fournir les services de conseil. A. The CAE should begin performing consulting services.
B. Le RAI devrait obtenir l’approbation du Comité d’Audit. B. The CAE should get approval from the audit committee.
C. La Charte de l’audit interne devrait être modifiée. C. The internal audit charter should be amended.
D. Le Conseil d’Administration devrait établir les politiques et les procédures D. The board should develop appropriate policies and procedures
adéquates pour pouvoir mener de telles missions. for conducting such engagements.
La réponse (A) est fausse. Après avoir obtenu l’approbation du Conseil Answer (A) is incorrect because, after the CAE gets board
d’administration, la charte de l’audit interne devra être modifiée et l’activité de approval, the audit charter should be amended and the internal
l’audit interne devra définir les politiques et les procédures appropriées pour audit activity should develop appropriate policies and procedures
mener de telles missions. for conducting such engagements.
La réponse (B) est fausse. Le RAI n’a pas besoin d’obtenir l’approbation du Answer (B) is incorrect because the CAE does not need to get
comité d’audit. L’approbation du conseil d’administration suffit. additional approval from the audit committee; only board approval
La réponse (C) est juste. De temps à autre on demande aux auditeurs is required.
internes d’effectuer des services de conseil dans des domaines où Answer (C) is correct. Internal auditors are sometimes
précédemment ils ont eu des positions de responsabilité ou pour requested to provide consulting services relating to
lesquels ils ont conduit une mission d’assurance. Avant de proposer un operations for which they had previous responsibilities or had
service de conseil, le RAI devra s’assurer que le Conseil d’Administration conducted assurance services. Prior to offering consulting
comprenne et donne son approbation au concept de fournir des services, the Chief Audit Executive should confirm that the
prestations de conseil. Une fois cette approbation obtenue la charte de board understands and approves the concept of providing
l’audit interne devra être modifiée afin d’inclure l’autorisation et les consulting services. Once approved, the internal audit charter
responsabilités pour les activités de conseil et le département d’audit should be amended to include authority and responsibilities
interne devra définir les politiques et les procédures nécessaires pour for consulting activities, and the internal audit activity should
mener de telles missions. (CRIPP) develop appropriate policies and procedures for conducting
La réponse (D) est fausse. L’audit interne devra définir les politiques et les such engagements. (IPPF)
procédures nécessaires pour mener de telles missions. Answer (D) is incorrect because the internal audit activity should
develop appropriate policies and procedures for conducting such
engagements.
Restitution
La réponse (A) est fausse. Ces Normes définissent les normes de Answer (A) is incorrect because they apply to Standards for
fonctionnement des organisations et des individus réalisant des activités Performance of organizations and individuals performing internal
d'audit interne. auditing services.
La réponse (B) est fausse. Ces Normes définissent les normes de qualification Answer (B) is incorrect because they apply to Standards for
des organisations et des individus réalisant des activités d'audit interne. Attributes of organizations and individuals performing internal
La réponse (C) est juste. D'après le Cadre de référence des pratiques auditing services.
professionnelles, les Normes pour la pratique professionnelle de l'audit Answer (C) is correct. According to the Professional Practices
interne - Normes de mise en oeuvre s'appliquent aux organisations et Framework, Standards for the Professional Practice of
aux personnes réalisant des types de mission d'audit interne Internal Auditing - Implementation Standards apply to
spécifiques définies dans ces Normes, à savoir essentiellement des organizations and individuals performing the specific internal
missions d'assurance et de conseil. Elles se divisent en deux grandes auditing services delineated by these Standards. Board
catégories : les Normes de mise en oeuvre pour les missions categories are assurance services and consulting services
d'assurance et celles pour les missions de conseil. Implementation Standards.
La réponse (D) est fausse, car seules les Normes de mise en oeuvre Answer (D) is incorrect because only Implementation standards
s'appliquent à des organisations et personnes réalisant des catégories de apply to organizations and individuals performing the specific
missions d'audit interne spécifiques définies dans ces Normes. internal auditing services delineated by these Standards.
P1 - 17 -- Un service d'audit interne a planifié une mission portant sur un P1 - 17 -- An internal audit activity has scheduled an engagement
contrat de BTP. Une partie de cette mission consistera à comparer les relating to a construction contract. One portion of this engagement
matériaux achetés à ceux spécifiés dans les plans et dessins techniques. Le will include comparing materials purchased with those specified in
service de l'audit interne ne dispose pas d'une personne ayant des the engineering drawings. The IAA does not have anyone on staff
compétences suffisantes pour mener à bien cette tâche. Son responsable with sufficient expertise to complete this procedure. The chief audit
doit donc : executive should
A. Annuler la mission. A. Delete the engagement from the schedule.
B. Faire appel au personnel existant pour réaliser l'ensemble de la mission. B. Perform the entire engagement using current staff.
C. Recourir à un consultant en ingénierie pour qu'il procède à la comparaison C. Engage an engineering consultant to perform the comparison.
nécessaire. D. Accept the contractor's written representations.
D. Accepter les déclarations écrites de l'entrepreneur.
La réponse (A) est fausse. Cette mission fait partie du champ de l'audit interne. Answer (A) is incorrect because the engagement is within the scope of
La réponse (B) est fausse. Il n'est pas approprié de faire appel au personnel the IAA.
existant (non qualifié) pour réaliser cette mission. Answer (B) is incorrect because performing the engagement using
La réponse (C) est juste. Le service d'audit interne doit disposer d'un the current (unqualified) staff is inappropriate.
personnel qualifié ou recourir à des prestataires extérieurs qualifiés Answer (C) is correct. The IAA should have employees or use
dans des disciplines telles que la comptabilité, l'audit, l'économie, la outside service providers who are qualified in such
finance, les statistiques, les technologies de l'information, l'ingénierie, disciplines as accounting, auditing, economics, finance,
la fiscalité, le droit ou l'environnement, et dans d'autres domaines, afin statistics, information technology, engineering, taxation, law,
de pouvoir exercer ses responsabilités. Chaque membre de ce service environmental affairs, and such other areas as needed to
n'a cependant pas besoin d'être qualifié dans toutes ces disciplines meet the IAA's responsibilities. Each member of the IAA,
(CRIPP). Par conséquent, pour faire preuve de conscience however, need not be qualified in all of these disciplines
professionnelle, il est nécessaire de recourir à un prestataire extérieur. (IPPF). Thus, hiring an outside service provider is also
La réponse (D) est fausse. Il n'est pas approprié d'accepter les déclarations necessary as an exercise of due professional care.
écrites de l'entrepreneur sans procéder à une vérification adéquate. Answer (D) is incorrect because accepting the contractor's
representations without adequate testing is inappropriate.
Restitution
Restitution
La réponse (A) est fausse, car tous les éléments cités sont des tâches que Answer (A) is incorrect because all of the items are things that the
l'auditeur interne doit exécuter afin de mener à bien une mission de conseil internal auditor should do when performing a formal consulting
formelle, outre les conditions d'indépendance et d'objectivité ainsi que la engagement in addition to the independence and objectivity
conscience professionnelle requise. evaluation and due professional care considerations.
La réponse (B) est fausse, car tous les éléments cités sont des tâches que Answer (B) is incorrect because all of the items are things that the
l'auditeur interne doit exécuter afin de mener à bien une mission de conseil internal auditor should do when performing a formal consulting
formelle, outre les conditions d'indépendance et d'objectivité ainsi que la engagement in addition to the independence and objectivity
conscience professionnelle requise. evaluation and due professional care considerations.
La réponse (C) est fausse, car tous les éléments cités sont des tâches que Answer (C) is incorrect because all of the items are things that the
l'auditeur interne doit exécuter afin de mener à bien une mission de conseil internal auditor should do when performing a formal consulting
formelle, outre les conditions d'indépendance et d'objectivité ainsi que la engagement in addition to the independence and objectivity
conscience professionnelle requise. evaluation and due professional care considerations.
La réponse (D) est juste. Outre les conditions d'indépendance et Answer (D) is correct. In addition to the independence and
d'objectivité ainsi que la conscience professionnelle requise, l'auditeur objectivity evaluation and due professional care
interne doit : organiser les réunions appropriées et recueillir les considerations, the internal auditor should: Conduct
informations nécessaires pour évaluer la nature et l'étendue des appropriate meetings and gather necessary information to
prestations à fournir ; s'assurer que les bénéficiaires des prestations assess the nature and extent of the service to be provided.
adhèrent aux dispositions de la Charte d'audit interne, aux règles et Confirm that those receiving the service understand and
procédures d'audit interne et aux autres dispositions régissant la agree with the relevant guidance contained in the internal
conduite des missions de conseil. L'auditeur interne doit refuser les audit charter, internal audit activity's policies and procedures,
missions de conseil qui sont prohibées par la Charte d'audit interne, and other related guidance governing the conduct of
qui sont contraires aux règles et procédures du service d'audit interne, consulting engagements. The internal auditor should decline
ou qui n'apportent pas de valeur ajoutée ni ne répondent aux intérêts to perform consulting engagements that are prohibited by the
de l'organisation ; s'assurer de la compatibilité de la mission de conseil terms of the internal audit charter, conflict with the policies
avec le plan d'audit. Ce plan, fondé sur une analyse des risques, peut and procedures of the internal audit activity, or do not add
intégrer et s'appuyer sur des missions de conseil, dans la mesure où value and promote the best interests of the organization.
elles contribuent à assurer la couverture d'audit nécessaire à Evaluate the consulting engagement for compatibility with the
l'organisation ; formaliser, dans un accord ou plan écrit, les termes internal audit activity's overall plan of engagements. The
généraux, la compréhension, les produits attendus et les autres internal audit activity's risk-based plan of engagements may
facteurs clés de la mission de conseil. Il est essentiel que l'auditeur incorporate and rely on consulting engagements, to the
interne et les bénéficiaires des prestations de conseil définissent d'un extent deemed appropriate, to provide necessary audit
commun accord les exigences en termes de rapport et de coverage to the organization. Document general terms,
communication. (CRIPP) understandings, deliverables, and other key factors of the
formal consulting engagement in a written agreement or plan.
It is essential that both the internal auditor and those
receiving the consulting engagement understand and agree
with the reporting and communication requirements. (IPPF)
Restitution
La réponse (A) est fausse. L'auditeur ne fait pas de la rétention d'information. Answer (A) is incorrect. The auditor is not withholding information
L'information a en effet bien été communiquée au responsable de l'audit because the information has been forwarded to the chief audit
interne. Elle peut se révéler utile lors d'une mission ultérieure dans le service executive. The information may be useful in a subsequent
marketing. engagement in the marketing area.
La réponse (B) est fausse. L'auditeur a noté un signal d'alerte qui peut se Answer (B) is incorrect. The auditor has documented a red flag that
révéler important lors d'une mission ultérieure. Il n'enfreint pas les Normes. may be important in a subsequent engagement. This does not violate
La réponse (C) est juste. L'auditeur n'enfreint ni le Code de déontologie the Standards.
ni les Normes. Voir réponses A et B. Answer (C) is correct. There is no violation of either the Code
La réponse (D) est fausse. Voir réponses A et B. of Ethics or the Standards. See answers "a" and "b".
Answer (D) is incorrect. See answers "a" and "b".
P1 - 21 -- Laquelle des activités suivantes procure un retour d'information sur P1 - 21 -- Which of the following activities are designed to provide
l'efficacité d'un service d'audit interne ? feedback on the effectiveness of an internal audit function?
I. Une supervision appropriée. I. Proper supervision.
II. Une bonne formation. II. Proper training.
III. Des évaluations internes. III. Internal assessments.
IV. Des évaluations externes. IV. External assessments.
A. I, II et III uniquement. A. I, II, and III only.
B. I, II et IV uniquement. B. I, II, and IV only.
C. I, III et IV uniquement. C. I, III, and IV only.
D. II, III et IV uniquement. D. II, III, and IV only.
La réponse (A) est fausse. Une bonne formation est importante, mais elle ne Answer (A) is incorrect. Proper training is important but it does not
procure pas de retour d'information. provide feedback.
La réponse (B) est fausse. Une bonne formation est importante, mais elle ne Answer (B) is incorrect. Proper training is important but it does not
procure pas de retour d'information. provide feedback.
La réponse (C) est juste. Un programme d'assurance qualité est destiné Answer (C) is correct. Quality assurance programs are
à procurer un retour d'information sur l'efficacité de l'audit interne. Il designed to provide feedback on the effectiveness of an
doit inclure la supervision, qui apporte un retour d'information au jour internal audit function. A quality assurance program should
le jour. Un programme d'assurance qualité doit comporter des include supervision, which provides day-to-day feedback. A
évaluations internes et externes. quality assurance program should include internal and
La réponse (D) est fausse. Une bonne formation est importante, mais elle ne external assessments.
procure pas de retour d'information. Answer (D) is incorrect. Proper training is important but it does not
provide feedback.
Restitution
Restitution
P1 - 25 -- Le responsable de l'audit interne doit élaborer et tenir à jour un P1 - 25-- The chief audit executive should develop and maintain a
programme d'assurance et d'amélioration de la qualité qui couvre tous les quality assurance and improvement program that covers all
aspects de l'audit interne et effectue un suivi continu de son efficacité. Tous aspects of the internal audit activity and continuously monitors its
les éléments suivants sont compris dans un programme qualité, sauf effectiveness. All of the following are included in a quality program
except
A. L'évaluation annuelle des performances de chaque auditeur interne.
B. L'évaluation interne périodique. A. Annual appraisals of individual internal auditors' performance.
C. La supervision. B. Periodic internal assessment.
D. Les évaluations externes périodiques. C. Supervision.
D. Periodic external assessments.
La réponse (A) est juste. Un programme d'assurance et d'amélioration Answer (A) is correct. A quality assurance and improvement
qualité doit être conçu de manière à fournir aux différentes parties program should be designed to provide reasonable
prenantes de l'audit interne l'assurance raisonnable (1) qu'il respecte sa assurance to the various stakeholders of the IAA that it (1)
charte, qui doit elle-même se conformer aux Normes et au Code de performs in accordance with its charter, which should be
Déontologie, (2) qu'il est efficient et efficace et (3) qu'il est perçu par les consistent with the Standards and the Code of Ethics; (2)
parties prenantes comme contribuant à la création de valeur ajoutée et operates effectively and efficiently; and (3) is perceived by the
à l'amélioration du fonctionnement de l'organisation. Le programme stakeholders as adding value and improving operations. The
doit comporter une supervision adéquate, une évaluation interne program should include appropriate supervision, periodic
périodique et un suivi continu de l'assurance qualité, ainsi que des internal assessment and ongoing monitoring of quality
évaluations externes périodiques (CRIPP). Le programme de assurance, and periodic external assessments (IPPF). The
recrutement et de développement des ressources humaines de l'audit program for selecting and developing the human resources of
interne doit prévoir, entre autres, l'évaluation des performances de the IAA provides for, among other things, appraising each
chaque auditeur interne, au moins une fois par an (CRIPP). internal auditor's work at least annually (IPPF).
La réponse (B) est fausse. L'évaluation interne constitue un élément d'un Answer (B) is incorrect because internal assessment is an element
programme qualité. of a quality program.
La réponse (C) est fausse. La supervision constitue un élément d'un Answer (C) is incorrect because supervision is an element of a
programme qualité. Les examens continus sont des évaluations internes quality program. Ongoing reviews are internal assessments that
comprenant la supervision de la mission. include engagement supervision.
La réponse (D) est fausse. L'évaluation externe constitue un élément d'un Answer (D) is incorrect because external assessment is an
programme qualité. element of a quality program.
Restitution
PROBABILITÉ LIKELIHOOD
Lointaine Possible Probable Remote Possible Likely
IMPACT Critique Risque A Risque B IMPACT Critical Risk A Risk B
Majeur Risque D Major Risk D
Mineur Risque C Minor Risk C
Compte tenu des moyens limités dont dispose l’audit interne, quelle est la Which of the following is the correct prioritization of risks,
hiérarchie correcte des risques ? considering limited resources in the internal audit activity?
A. Risque B, risque C, risque A, risque D. A. Risk B, Risk C, Risk A, Risk D.
B. Risque A, risque B, risque C, risque D. B. Risk A, Risk B, Risk C, Risk D.
C. Risque D, risque B, risque A, risque C. C. Risk D, Risk B, Risk A, Risk C.
D. Risque B, risque C, risque D, risque A. D. Risk B, Risk C, Risk D, Risk A.
La réponse (A) est fausse. Le risque D serait prioritaire par rapport au risque A, en Answer (A) is incorrect. Risk D would take precedence over risk A,
raison de sa plus forte probabilité de survenue et malgré son moindre impact. as it has a higher probability of occurring despite the lower impact.
La réponse (B) est fausse. Ce n'est pas le bon ordre. Answer (B) is incorrect. This is not the correct order.
La réponse (C) est juste. Elle classifie les risques en combinant Answer (C) is correct. This order ranks the risk by a
probabilité et impact. combination of probability and impact.
La réponse (D) est fausse. Le risque D devrait être prioritaire par rapport au Answer (D) is incorrect. Risk D should be rated higher than risk C,
risque C, en raison de sa probabilité et de son impact supérieurs. due to probability and impact.
[Enoncé #20] Pendant la phase de planification, un responsable de l’audit [Fact Pattern #20] During the planning phase, a chief audit
interne évalue quatre missions d’après les facteurs suivants : la capacité à executive (CAE) is evaluating four audit engagements based on
atténuer le risque pour l’organisation, l’aptitude à préserver les ressources the following factors: the engagement’s ability to reduce risk to the
financières de l’organisation et l’ampleur des changements mis en œuvre organization, the engagement’s ability to save the organization
depuis la dernière mission. Pour chaque mission, le responsable de l’audit money, and the extent of change in the area since the last
interne assigne de 1 à 3 points par aspect examiné, et calcule un score engagement. The CAE has scored the engagements for each
global. Voici les résultats obtenus (nombre de points entre parenthèses) : factor from low to high, assigned points, and calculated an overall
ranking. The results are shown below with the points in
Atténuation du parenthesis:
Mission Économies Changements
risque
1 Forte (3) Moyennes (2) Peu nombreux (1) Audit Risk Reduction Cost Savings Changes
2 Forte (3) Faibles (1) Nombreux (3) 1 High (3) Medium (2) Low (1)
3 Faible (1) Fortes (3) Moyens (2) 2 High (3) Low (1) High (3)
4 Moyenne (2) Moyennes (2) Nombreux (3) 3 Low (1) High (3) Medium (2)
4 Medium (2) Medium (2) High (3)
P1 - 27 -- (Voir énoncé 20) Si l'organisation lui a demandé de considérer que
les économies sont deux fois plus importantes que tout autre facteur, quelles P1 - 27 -- (Refers to Fact Pattern #20) If the organization has asked
missions le responsable de l'audit interne doit-il continuer à mettre en œuvre? the CAE to consider the cost savings factor to be twice as
important as any other factor, which engagements should the CAE
A. 1 et 2 uniquement. pursue?
B. 1 et 3 uniquement.
C. 2 et 4 uniquement. A. 1 and 2 only.
D. 3 et 4 uniquement. B. 1 and 3 only.
C. 2 and 4 only.
La réponse (A) est fausse. Ce choix correspond au score total le moins élevé. D. 3 and 4 only.
La réponse (B) est fausse. Le score total est inférieur à celui des missions 3 et 4. Answer (A) is incorrect. This choice involves the least total
La réponse (C) est fausse. Le score total est inférieur à celui des missions 3 et 4. points.
La réponse (D) est juste. Ce choix correspond au score total le plus Answer (B) is incorrect. The total points are less than for engagements
élevé, et les missions concernées ont une capacité moyenne ou élevée 3 and 4.
à permettre des économies. Answer (C) is incorrect. The total points are less than for engagements
3 and 4.
Answer (D) is correct. This has the highest total points, and
the engagements have medium and high potentials for cost
savings.
Restitution
P1- 30 -- Le travail de l'audit interne comprend l’évaluation des P1- 30 -- The work of the internal audit activity includes
systèmes de management des risques et la participation à son evaluating and contributing to the improvement of risk
amélioration. Le risque est : management systems. Risk is
I. L'impact négatif d'événements dont on est sûr qu'ils se produiront. I. The negative effect of events certain to occur
II. Mesuré en termes de conséquences. II. Measured in terms of consequences
III. Mesuré en termes de probabilité. III. Measured in terms of likelihood
A. I uniquement.
B. I et II uniquement. A. I only.
C. II et III uniquement. B. I and II only.
D. I, II et III. C. II and III only.
D. I, II, and III.
La réponse (A) est fausse. Le risque suppose une notion d'incertitude
et les événements n'auront pas nécessairement d'impact négatif. Answer (A) is incorrect because risk involves uncertainty, and
La réponse (B) est fausse. Le risque suppose une notion d'incertitude the effects of events are not necessarily negative.
et les événements n'auront pas nécessairement d'impact négatif. Answer (B) is incorrect because risk involves uncertainty, and
La réponse (C) est juste. L'audit interne doit aider l'organisation the effects of events are not necessarily negative.
en identifiant et en évaluant les risques significatifs et contribuer Answer (C) is correct. The IAA should assist the
à l'amélioration des systèmes de management des risques et de organization by identifying and evaluating significant
contrôle (CRIPP). Le risque est la possibilité que se produise un exposures to risk and contributing to the improvement of
événement qui aura un impact sur la réalisation des objectifs. Il risk management and control systems (IPPF). Risk is the
se mesure en termes de conséquences et de probabilité uncertainty of an event the occurrence of which could
(Glossaire). have an effect on the achievement of organizational
La réponse (D) est fausse. Le risque suppose une notion d'incertitude objectives. It is measured in terms of consequences and
et les événements n'auront pas nécessairement d'impact négatif. likelihood (Glossary).
Answer (D) is incorrect because risk involves uncertainty, and
the effects of events are not necessarily negative.
Restitution
P1-32 -- Il y a pertinence des processus de management des risques, P1- 32 -- Adequacy of risk management, control, and
de contrôle et de gouvernement d’entreprise si le management les a governance processes is present if management has planned
conçus et organisés de telle manière qu’ils apportent une assurance and designed these processes in a manner that provides
raisonnable que les objectifs et les buts fixés seront efficacement reasonable assurance that the organization's objectives and
économiquement atteints. Laquelle des assertions suivantes n’est pas goals will be achieved efficiently and economically. Which of
vraie au regard de l’atteinte efficace et économique des objectifs et the following statements is not true regarding the efficient
des buts de l’organisation? and economical achievement of the organization's objectives
and goals?
A. Un fonctionnement économique permet d’atteindre les objectifs et
buts avec une utilisation minimale de ressources quels que soient les A. Economical performance accomplishes objectives and
risques encourus. goals with minimal use of resources with no regard to risk
B. Un fonctionnement efficient signifie que les buts et les objectifs sont exposure.
atteints à bonne date. B. Efficient performance accomplishes objectives and goals
C. Un fonctionnement économique permet d’atteindre les objectifs et in a timely manner.
buts avec une utilisation minimale de ressources en proportion du C. Economical performance accomplishes objectives and
risque encouru. goals with minimal use of resources commensurate with the
D. Un fonctionnement efficient permet d’atteindre avec précision les risk exposure.
objectifs et les buts fixés au moindre coût. D. Efficient performance accomplishes objectives and goals
in an accurate and economical manner.
La réponse (A) est correcte. Un fonctionnement efficace permet Answer (A) is correct. Efficient performance
d’atteindre avec précision les objectifs et les buts fixés, à bonne accomplishes objectives and goals in an accurate,
date et au moindre coût. Un fonctionnement économique permet timely, and economical fashion. Economical performance
d’atteindre les objectifs et buts avec une utilisation minimale des accomplishes objectives and goals with minimal use of
ressources (c’est à dire à des coûts moindres) en proportion du resources (i.e., cost) commensurate with risk exposure
risque encouru. Pour un fonctionnement économique selon les (IPPF). Thus, in order to achieve economical
objectifs et buts fixés, la réduction des coûts doit être rapprochée performance when accomplishing objectives and goals,
du degré d’exposition au risque. the minimal cost should correspond to the degree of risk
La réponse (B) est fausse. Cette assertion est vraie au regard de exposure.
l’atteinte efficace et économique des objectifs et des buts de Answer (B) is incorrect because this is a true statement
l’organisation. regarding the efficient and economic achievement of the
La réponse (C) est fausse. Cette assertion est vraie au regard de organization's objectives and goals.
l’atteinte efficace et économique des objectifs et des buts de Answer (C) is incorrect because this is a true statement
l’organisation. regarding the efficient and economic achievement of the
La réponse (D) est fauss. Cette assertion est vraie au regard de organization's objectives and goals.
l’atteinte efficace et économique des objectifs et des buts de Answer (D) is incorrect because this is a true statement
l’organisation. regarding the efficient and economic achievement of the
organization's objectives and goals.
Restitution
P1- 34 – Lequel (lesquels) des éléments suivants constitue(nt) des P1- 34 -- Which of the following are elements of risk
éléments relatifs à la gestion et à la réduction des risques ? management and mitigation?
I. Les analyses coût / bénéfice des menaces. I. Threat events and cost/benefit analysis.
II. Les protections, les contrôles, et les analyses du retour sur II. Safeguards, controls, and ROI analysis.
investissement. III. Frequency and uncertainty.
III. La fréquence et l’incertitude. IV. Safeguard and control costs.
IV. Les coûts de la protection et du contrôle.
A. I only.
A. I uniquement. B. II only.
B. II uniquement. C. I and III only.
C. I et III uniquement. D. II and IV only.
D. II et IV uniquement.
La réponse (A) est fausse. Les menaces sont des éléments relatifs à Answer (A) is incorrect because threat events is a risk
l’identification et à l’évaluation des risques. element associated with risk identification and quantification.
La réponse (B) est fausse. Les coûts de la protection et du contrôle Answer (B) is incorrect because safeguard and control costs--
(choix IV) constituent également des éléments relatifs à la gestion et à choice IV--is also a risk element that relates to risk
la réduction des risques. management and mitigation.
La réponse (C) est fausse. Les menaces, la fréquence et l’incertitude Answer (C) is incorrect because threat events, frequency,
sont tous des éléments relatifs à l’identification et à l’évaluation des and uncertainty are all risk elements associated with risk
risques. identification and quantification.
La réponse (D) est juste. Les éléments relatifs à la gestion et à la Answer (D) is correct. The elements relating to risk
réduction des risques comprennent les protections et les management and mitigation include safeguards and
contrôles, les coûts de la protection et du contrôle, et les controls, safeguard and control costs, and cost/benefit
analyses coût / bénéfice et retour sur investissement. Les autres or ROI analysis. The other elements of risk-threat events,
éléments liés aux menaces, l’impact, la fréquence et l’incertitude single loss exposure value, frequency, and uncertainty-
font partie de l’identification et à l’évaluation des risques. comprise risk identification and quantification.
Restitution
Restitution
Restitution
La réponse (A) est juste. Pour formuler une opinion globale sur Answer (A) is correct. Internal auditors must determine
l'adéquation du dispositif de management des risques, les that the organization's risk management processes
auditeurs internes doivent également s'assurer que le processus address five key objectives to formulate an opinion on
de management des risques de l'organisation répond à cinq the overall adequacy of the risk management processes.
objectifs principaux. Les cinq principaux objectifs d'un dispositif The five key objectives of a risk management process
de management des risques sont les suivants : les risques are: Risks arising from business strategies and activities
découlant des stratégies et des activités de l'organisation sont are identified and prioritized. Management and the board
identifiés et hiérarchisés ; le management et le Conseil ont have determined the level of risks acceptable to the
déterminé un niveau de risques acceptable pour l'organisation, en organization, including the acceptance of risks designed
tenant compte des risques liés à la mise en oeuvre des plans to accomplish the organization's strategic plans. Risk
stratégiques de l'organisation ; des mesures d'atténuation des mitigation activities are designed and implemented to
risques sont définies et mises en place afin de réduire ou de gérer reduce, or otherwise manage, risk at levels that were
les risques, compte tenu des seuils jugés acceptables par le determined to be acceptable to management and the
management et le Conseil ; un suivi permanent des activités est board. Ongoing monitoring activities are conducted to
effectué afin de réévaluer périodiquement les risques et periodically reassess risk and the effectiveness of
l'efficacité des contrôles permettant de les gérer ; des rapports controls to manage risk. The board and management
concernant les résultats des processus de management des receive periodic reports of the results of the risk
risques sont adressés périodiquement au Conseil et au management processes. The corporate governance
management. Le processus de gouvernement d'entreprise doit processes of the organization should provide periodic
fournir une présentation périodique des risques, des stratégies communication of risks, risk strategies, and controls to
liées aux risques, et des contrôles, destinée aux parties stakeholders.
prenantes de l'organisation. Answer (B) is incorrect because review of previous risk
La réponse (B) est fausse. L'examen des précédents rapports evaluation reports by management, internal auditors, external
d'évaluation des risques par la direction, les auditeurs internes, les auditors, and any other sources that may have issued such
auditeurs externes et toutes autres sources qui peuvent avoir publié ce reports is an engagement procedure, not a key objective, that
type de rapports constitue une procédure de mission, et non un objectif the internal auditor should consider when gathering sufficient
clé, que l'auditeur interne doit prendre en compte lorsqu'il collecte information regarding the five key objectives of the risk
suffisamment d'informations sur les cinq objectifs clés du processus de management process.
gestion des risques. Answer (C) is incorrect because review of previous risk
La réponse (C) est fausse. L'examen des précédents rapports evaluation reports by management, internal auditors, external
d'évaluation des risques par la direction, les auditeurs internes, les auditors, and any other sources that may have issued such
auditeurs externes et toutes autres sources qui peuvent avoir publié ce reports is an engagement procedure, not a key objective, that
type de rapports constitue une procédure de mission, et non un objectif the internal auditor should consider when gathering sufficient
clé, que l'auditeur interne doit prendre en compte lorsqu'il collecte information regarding the five key objectives of the risk
suffisamment d'informations sur les cinq objectifs clés du processus de management process.
gestion des risques. Answer (D) is incorrect because review of previous risk
La réponse (D) est fausse. L'examen des précédents rapports evaluation reports by management, internal auditors, external
d'évaluation des risques par la direction, les auditeurs internes, les auditors, and any other sources that may have issued such
auditeurs externes et toutes autres sources qui peuvent avoir publié ce reports is an engagement procedure, not a key objective, that
type de rapports constitue une procédure de mission, et non un objectif the internal auditor should consider when gathering sufficient
clé, que l'auditeur interne doit prendre en compte lorsqu'il collecte information regarding the five key objectives of the risk
suffisamment d'informations sur les cinq objectifs clés du processus de management process.
gestion des risques.
Restitution
Restitution
P1- 41 -- Lesquels des éléments suivants font partie de P1- 41 -- Which of the following are elements included in the
l'environnement de contrôle ? control environment?
A. La structure organisationnelle, la philosophie de direction et la A. Organizational structure, management philosophy, and
planification. planning.
B. L'intégrité et les valeurs éthiques, l'attribution des pouvoirs, les B. Integrity and ethical values, assignment of authority, and
politiques relatives aux ressources humaines. human resource policies.
C. La compétence du personnel, les installations de sauvegarde, le C. Competence of personnel, backup facilities, laws, and
droit et la réglementation. regulations.
D. L'évaluation des risques, l'attribution des responsabilités et les D. Risk assessment, assignment of responsibility, and human
pratiques relatives aux ressources humaines. resource practices.
La réponse (A) est fausse. La planification ne fait pas partie de Answer (A) is incorrect because planning is not an element of
l'environnement de contrôle. the control environment.
La réponse (B) est juste. D'après le Glossaire annexé aux Normes, Answer (B) is correct. According to the Glossary
l'environnement de contrôle inclut « l'attitude et les actions du appended to the Standards, the control environment
Conseil et du management au regard de l'importance du contrôle includes the attitude and actions of the board and
dans l'organisation. L'environnement de contrôle constitue le management regarding the significance of control within
cadre et la structure nécessaires à la réalisation des objectifs the organization. The control environment provides the
primordiaux du système de contrôle interne. L'environnement de discipline and structure for the achievement of the
contrôle englobe les éléments suivants : intégrité et valeurs primary objectives of the system of internal control. The
éthiques, philosophie et style de direction, structure control environment includes the following elements:
organisationnelle, attribution des pouvoirs et responsabilités, Integrity and ethical values Management's philosophy
politiques et pratiques relatives aux ressources humaines, and operating style Organizational structure Assignment
compétence du personnel ». of authority and responsibility Human resource policies
La réponse (C) est fausse. Les installations de sauvegarde, le droit et and practices Competence of personnel
la réglementation ne font pas partie de l'environnement de contrôle. Answer (C) is incorrect because backup facilities, laws, and
La réponse (D) est fausse. L'évaluation des risques fait partie de la regulations are not elements of the control environment.
planification de l'audit interne et des missions spécifiques. Answer (D) is incorrect because risk assessment is part of
planning the internal audit activity and specific engagements.
Restitution
P1- 43 -- Le contrôle peut être décrit comme un système fermé P1- 43 -- Control may be described as a closed system
comportant six éléments. L'un de ces éléments est : consisting of six elements. One of the six elements is
A. L'instauration d'objectifs de performance. A. Setting performance standards.
B. La sécurisation adéquate de fichiers de données. B. Adequately securing data files.
C. L'approbation de la charte d'audit interne. C. Approval of the internal audit activity's charter.
D. L'instauration d'un service d'audit interne. D. Establishment of an independent internal audit activity.
La réponse (A) est juste. D'après Internal Auditing de Sawyer (IIA Answer (A) is correct. Sawyer's Internal Auditing (IIA
1988, p. 979 de l'original), ouvrage sur lequel se fonde cette 1988, p. 979), on which this question is based, describes
question, le système de contrôle comporte six éléments : (1) la a six-element control system: (1) setting standards, (2)
définition d'objectifs de performance, (2) la mesure de la measuring performance, (3) analyzing performance and
performance, (3) l'analyse de la performance et la comparaison comparing it with the standards, (4) evaluating deviations
avec les objectifs, (4) l'évaluation des écarts, qui doivent être and bringing them to the attention of appropriate
portés à l'attention des personnes concernées, (5) la correction persons, (5) correcting deviations, and (6) following up
de ces écarts, et (6) le suivi des actions correctives. on corrective action.
La réponse (B) est fausse. La sécurisation adéquate de fichiers de Answer (B) is incorrect because adequately securing data
données ne constitue pas l'un des éléments d'un système de contrôle files is not an element of a closed control system.
fermé. Answer (C) is incorrect because approval of the IAA's charter
La réponse (C) est fausse, l'approbation de la charte d'audit interne ne is not an element of a closed control system.
constitue pas l'un des éléments d'un système de contrôle fermé. Answer (D) is incorrect because establishment of an
La réponse (D) est fausse. L'établissement d'un service d'audit interne independent IAA is not an element of a closed control
ne constitue pas l'un des éléments d'un système de contrôle fermé. system.
Restitution
A. Les procédures d'achat sont bien conçues et respectées, sauf A. Purchasing procedures are well designed and are followed
lorsque le superviseur des achats exige qu'il en soit autrement. unless otherwise directed by the purchasing supervisor.
B. Des bons de commande vierges prénumérotés sont rangés en lieu B. Prenumbered blank purchase orders are secured within
sûr dans le service achats. the purchasing department.
C. Les achats opérationnels courants varient de 500 à 1 000 dollars, C. Normal operational purchases fall in the range from $500
deux signatures étant nécessaires pour les achats supérieurs à 1 000 to $1,000 with two signatures required for purchases over
dollars. $1,000.
D. L'acheteur investit dans une SICAV négociée sur le marché qui D. The purchasing agent invests in a publicly traded mutual
intègre le titre de l'un des fournisseurs de l'organisation dans son fund that lists the stock of one of the organization's suppliers
portefeuille. in its portfolio.
La réponse (A) est juste. Des procédures bien conçues qui sont Answer (A) is correct. Well-designed procedures that are
laissées de côté au gré du personnel dirigeant ne caractérisent set aside at management's discretion are not adequate
pas des contrôles adéquats. Pour être efficaces, les procédures controls. Control procedures must be followed
de contrôle doivent être respectées de manière systématique. consistently to be effective. However, the possibility of
Cependant, la possibilité pour la direction de passer outre management override is an inherent limitation of internal
constitue une limitation inhérente au contrôle interne. control.
La réponse (B) est fausse. L'utilisation de bons de commande vierges Answer (B) is incorrect because use of prenumbered blank
prénumérotés rangés en lieu sûr dans le service achats constitue un purchase orders secured within the purchasing department is
contrôle ordinaire. a common control.
La réponse (C) est fausse. Le fait d'exiger une procédure d'autorisation Answer (C) is incorrect because requiring a more stringent
plus stricte pour des achats plus importants constitue un contrôle authorization procedure for larger purchases is an
approprié tant que des pièces justificatives sont fournies à l'appui de appropriate control as long as documentation supports the
ces achats. purchases.
La réponse (D) est fausse. La SICAV retenue par l'acheteur ne doit Answer (D) is incorrect because the purchasing agent's
pas constituer un conflit d'intérêts. Or, la relation entre le retour sur mutual fund investment should not be a conflict of interest.
investissement et d'éventuelles mesures qui pourraient être prises par The relationship between the return on the investment and
l'acheteur pour favoriser le fournisseur est très ténue. any possible action by the agent to favor the supplier is very
weak.
P1- 45 -- L’auditeur interne sait que tout dispositif de contrôle interne a P1- 45 -- The internal auditor recognizes that certain
ses limites. Lequel des scénarios suivants est le résultat d’une limite limitations are inherent in any system of internal controls.
inhérente au dispositif de contrôle interne ? Which one of the following scenarios is the result of an
inherent limitation of internal control?
A. Le contrôleur financier réalise à la fois les dépôts d’argent liquide et A. The comptroller both makes and records cash deposits.
leur enregistrement. B. A security guard allows one of the warehouse employees
B. Un agent de sécurité autorise un des employés de l’entrepôt à sortir to remove assets from the premises without authorization.
des biens des locaux sans autorisation. C. The organization sells to customers on account, without
C. L’organisation vend à crédit à ses clients sans approbation du credit approval.
crédit. D. An employee, who is unable to read, is assigned custody
D. Un employé, qui ne sait pas lire, est affecté à la surveillance de la of the organization's computer tape library and run manuals
bandothèque de l’organisation et déroule des procédures qui sont that are used during the third shift.
utilisées au cours de la vacation de nuit.
La réponse (A) est fausse. La séparation de fonctions entre Answer (A) is incorrect because segregating the functions of
l’enregistrement et la conservation des biens est habituelle. Le fait que recording and asset custody is customary. That the
le contrôleur financier réalise à la fois les dépôts d’argent liquide et leur comptroller both makes and records cash deposits is an
enregistrement est une faiblesse de contrôle qui aurait pu être évitée. avoidable control weakness.
La réponse (B) est juste. Les limites inhérentes au contrôle Answer (B) is correct. Inherent limitations in internal
interne surviennent en cas d’erreurs de jugement, control arise from mistakes in judgment,
d’incompréhension des instructions, d’un manque d’attention du misunderstandings of instructions, personnel
personnel, de distraction, de fatigue, de collusion, d’actes en carelessness, distraction, fatigue, collusion,
provenance de la direction, de modifications du contexte, et de perpetrations by management, changing conditions, and
détérioration des niveaux de conformité. En conséquence, un deterioration of degrees of compliance. Thus, a control
contrôle (le recours à des agents de sécurité) basé sur la (use of security guards) based on segregation of
séparation de fonctions peut être contourné par la collusion entre functions may be overcome by collusion among two or
deux employés ou plus. more employees.
La réponse (C) est fausse. Les transactions peuvent et devraient être Answer (C) is incorrect because transactions can and should
autorisées avant leur exécution. be authorized before execution.
La réponse (D) est fausse. L’affectation de personnel non qualifié est Answer (D) is incorrect because assignment of an unqualified
une faiblesse de contrôle qui aurait pu être évitée. employee is an avoidable control weakness.
Restitution
P1- 47 -- Un processus de contrôle type comporte les étapes P1- 47 -- The steps in a typical control process include:
suivantes:
1. Selecting strategic control points at which to gather
1. Sélection des points de contrôle stratégiques au niveau desquels information about activities being performed
seront recueillies des informations sur les activités en cours. 2. Accumulating, classifying, and recording data samples
2. Accumulation, classification et enregistrement d'échantillons de 3. Observing the work or collecting samples of data
données. 4. Determining whether performance is satisfactory
3. Observation du travail ou collecte d'échantillons de données. 5. Reviewing and revising standards
4. Vérification du degré satisfaisant des performances. 6. Reporting significant deviations to managers concerned
5. Examen et révision des normes.
6. Notification des écarts significatifs aux cadres concernés.
Dans quel ordre ces étapes doivent-elles se succéder ? What is the proper order of these steps?
A. 1, 3, 2, 4, 6, 5. A. 1, 3, 2, 4, 6, 5.
B. 1, 2, 3, 4, 5, 6. B. 1, 2, 3, 4, 5, 6.
C. 1, 3, 4, 2, 6, 5. C. 1, 3, 4, 2, 6, 5.
D. 1, 3, 4, 2, 5, 6. D. 1, 3, 4, 2, 5, 6.
La réponse (A) est juste. La succession appropriée des étapes Answer (A) is correct. The proper sequence of steps in a
d'un processus de contrôle type est la suivante : typical control process is as follows:
1. Sélection des points de contrôle stratégiques au niveau 1. Selecting strategic control points at which to gather
desquels seront recueillies des informations sur les activités en information about activities being performed
cours de réalisation. 2. Observing the work or collecting samples of data
2. Observation du travail ou collecte d'échantillons de données. 3. Accumulating, classifying, and recording data samples
3. Accumulation, classification et enregistrement d'échantillons 4. Comparing samples with predetermined quality,
de données. schedule, and cost standards
4. Comparaison des échantillons avec les normes prédéfinies 5. Determining whether performance is satisfactory
relatives à la qualité, au calendrier et aux coûts. 6. Reporting significant deviations to managers
5. Vérification du degré satisfaisant des performances. concerned
6. Notification des écarts significatifs aux cadres concernés. 7. Determining, by repeating the above steps, whether
7. Vérification, par la répétition des étapes ci-dessus, de action taken is effective in correcting reported deviations
l'efficacité des dispositions prises pour corriger les écarts (follow-up)
signalés (suivi). 8. Reviewing and revising standards
8. Examen et révision des normes. Answer (B) is incorrect because observation must occur
La réponse (B) est fausse. L'observation doit précéder la classification before classifying and recording data samples.
et l'enregistrement des échantillons de données. Answer (C) is incorrect because data must be recorded
La réponse (C) est fausse. Les données doivent être enregistrées before comparisons can occur.
avant de pouvoir être comparées. Answer (D) is incorrect because data must be recorded
La réponse (D) est fausse. Les données doivent être enregistrées before comparisons can occur.
avant de pouvoir être comparées.
Restitution
P1- 49 -- Laquelle des affirmations suivantes sur le gouvernement P1- 49 -- Which of the following statements regarding
d'entreprise n'est pas correcte ? corporate governance is not correct?
A. Les mécanismes de contrôle des entreprises peuvent être internes A. Corporate control mechanisms include internal and
et externes. external mechanisms.
B. La grille de rémunération de la direction fait partie des mécanismes B. The compensation scheme for management is part of the
de contrôle des entreprises. corporate control mechanisms.
C. La dilution de la richesse des actionnaires dues aux stock-options C. The dilution of shareholders' wealth resulting from
ou aux primes en actions accordées aux salariés relève davantage de employee stock options or employee stock bonuses is an
la comptabilité que du gouvernement d'entreprise. accounting issue rather than a corporate governance issue.
D. L'auditeur interne d'une entreprise a davantage de responsabilités D. The internal auditor of a company has more responsibility
que le conseil d'administration dans le domaine du gouvernement than the board for the company's corporate governance.
d'entreprise.
La réponse (A) est fausse. Les mécanismes de contrôle des Answer (A) is incorrect. Corporate control mechanisms do
entreprises sont bien internes et externes. include internal and external mechanisms.
La réponse (B) est fausse. La grille de rémunération de la direction fait Answer (B) is incorrect. Management's compensation
bien partie des mécanismes de contrôle des entreprises. scheme is part of corporate control mechanisms.
La réponse (C) est fausse. La dilution de la richesse des actionnaires Answer (C) is incorrect. The dilution of shareholder's wealth
dues aux stock-options ou aux primes en actions accordées aux resulting from employee stock options or employee stock
salariés relève effectivement davantage de la comptabilité que du bonuses is an accounting issue rather than a corporate
gouvernement d'entreprise. governance issue.
La réponse (D) est juste. C'est le conseil d'administration, et non Answer (D) is correct. The board is ultimately
les auditeurs internes, qui est responsable en dernier ressort du responsible for the company's corporate governance, not
gouvernement d'entreprise. the internal auditors.
Restitution
Tous ces départements, sauf deux, figurent sur la liste des clients de All of these departments, except two, are on the potential list
mission potentiels suite à une analyse des risques par le responsable of engagement clients because of a risk analysis performed
de l’audit interne. Le service de production A est sur la liste parce que by the chief audit executive. Production department A is on
le président pense qu’il existe trop de goulets d’étranglement dans ce the list because the president thinks too many bottlenecks
département. Le service marketing est sur la liste parce que le chef de occur in that department. The marketing department is on the
la sécurité a reçu un coup de téléphone anonyme qui affirmait que l’un list because the chief of security received an anonymous
des responsables du marketing acceptait d’importants pots de vin de la phone call accusing a marketing manager of accepting
part d’un organe de presse. Les contrôles internes semblent adéquats substantial financial kickbacks from a media outlet. Internal
dans tous les services, à l’exception éventuellement du marketing. controls seem adequate in all departments, with the possible
exception of marketing.
P1- 50 -- (Voir énoncé 14) Quelle est la définition la plus logique du P1- 50 -- (Refers to Fact Pattern #14) What is the chief audit
risque de perte que le responsable de l'audit interne utilise lorsqu'il executive's most logical definition of risk of loss to be used in
sélectionne les clients de la mission ? selecting engagement clients?
A. Le degré d'exposition au risque multiplié par la probabilité de perte. A. Amount of risk exposure times the probability of loss.
B. Le montant des coûts annuels dans un service. B. Amount of annual costs in a department.
C. La probabilité de perte. C. Probability of loss.
D. La quantité d'actifs au sein d'un service. D. Amount of assets in a department.
La réponse (A) est juste. Pour faciliter l'établissement des ordres Answer (A) is correct. To facilitate development of an
de priorité, le responsable de l'audit interne doit s'appuyer sur « engagement work schedule, the CAE should consider
une évaluation à jour des risques et de l'efficacité des dispositifs updated assessments of risks and the effectiveness of
de management des risques et de contrôle » (CRIPP). Le risque risk management and control processes (IPPF). Risk is
est la « possibilité que se produise un événement qui aura un the uncertainty of the occurrence of an event that could
impact sur la réalisation des objectifs. Le risque se mesure en affect the achievement of the organization's objectives. It
termes de conséquences et de probabilité » (glossaire annexé is measured in terms of consequences and likelihood
aux Normes). Ainsi, le risque de perte se définit logiquement (Glossary appended to the Standards). Thus, risk of loss
comme une valeur attendue. Il est égal au montant à risque is most logically defined as an expected value. It equals
multiplié par la probabilité de perte. the amount at risk times the probability of loss.
La réponse (B) est fausse. Le montant des coûts dans un département Answer (B) is incorrect because the amount of costs in a
n'est pas nécessairement équivalent au montant exposé à un risque department is not necessarily the amount exposed to a risk of
de perte. loss.
La réponse (C) est fausse. La probabilité de perte doit être multipliée Answer (C) is incorrect because the probability of a loss must
par le montant exposé à un risque de perte. be multiplied by the amount exposed to possible loss.
La réponse (D) est fausse. La quantité d'actifs dans un service n'est Answer (D) is incorrect because the amount of assets in a
pas nécessairement équivalante au montant exposé à un risque de department is not necessarily the amount exposed to a risk of
perte. loss.
Restitution
P1 - 52 -- La vision qu'a le management du rôle de l'audit interne sera P1 - 52 -- Management's view of the internal audit activity's role is
vraisemblablement déterminée par tous les facteurs suivants sauf : likely to be determined by all of the following factors except:
A. La culture de l'organisation. A. Organizational culture.
B. Les préférences de l'auditeur indépendant. B. Preferences of the independent auditor.
C. La capacité de l'équipe d'audit interne. C. Ability of the internal auditing staff.
D. La situation et les usages du pays. D. Local conditions and customs of the country.
La réponse (A) est fausse. C'est un facteur qui influence la vision du Answer (A) is incorrect because this is a factor that influences
management quant au rôle de l'audit interne. management's view on the role of internal audit.
La réponse (B) est juste. Il revient aux cadres dirigeants et au comité Answer (B) is correct. It is the role of executive management
d'audit de déterminer le rôle de l'audit interne dans le processus de and the audit committee to determine the role of internal audit
management des risques. La vision du management dans ce domaine in the risk management process. Management's view on
sera vraisemblablement fonction de facteurs tels que la culture de internal audit's role is likely to be determined by factors such
l'organisation, la compétence des auditeurs internes, la situation et les as the culture of the organization, ability of the internal
usages du pays (CRIPP). auditing staff, and local conditions and customs of the
La réponse (C) est fausse. C'est un facteur qui influence la vision du country (IPPF).
management quant au rôle de l'audit interne. Answer (C) is incorrect because this is a factor that influences
La réponse (D) est fausse. C'est un facteur qui influence la vision du management's view on the role of internal audit.
management quant au rôle de l'audit interne. Answer (D) is incorrect because this is a factor that influences
management's view on the role of internal audit.
P1 - 53 -- Les détails du programme d'audit servant à auditer les activités de P1 - 53 -- The details of the audit program used to audit e-
commerce électronique dans des organisations spécifiques varient en commerce activities in specific organizations will vary depending on
fonction de tous les facteurs suivants, à l'exception d'un, lequel ? all but which of the following factors?
A. Le secteur d'activité. A. Industry.
B. La culture de l'organisation. B. Organizational culture.
C. Le pays. C. Country.
D. Les modèles juridiques et d'entreprise. D. Legal and business models.
La réponse (A) est fausse. Il s'agit d'un facteur dont dépendent les détails du Answer (A) is incorrect because this is a factor upon which the
programme d'audit servant à vérifier les activités de commerce électronique. details of the audit program used to audit e-commerce activities are
La réponse (B) est juste. Les détails du programme d'audit servant à dependent.
vérifier les activités de commerce électronique dans des organisations Answer (B) is correct. The details of the audit program used to
spécifiques varient en fonction du secteur d'activité, du pays et des audit e-commerce activities in specific organizations will vary
modèles juridiques et d'entreprise. depending on industry, country, and legal and business
La réponse (C) est fausse. Il s'agit d'un facteur dont dépendent les détails du models.
programme d'audit servant à vérifier les activités de commerce électronique. Answer (C) is incorrect because this is a factor upon which the
La réponse (D) est fausse. Il s'agit d'un facteur dont dépendent les détails du details of the audit program used to audit e-commerce activities are
programme d'audit servant à vérifier les activités de commerce électronique. dependent.
Answer (D) is incorrect because this is a factor upon which the
details of the audit program used to audit e-commerce activities are
dependent.
Restitution
La réponse (A) est fausse. Ce point serait pris en compte lors de la définition Answer (A) is incorrect. This factor would be considered in
de l'ordre de priorité des missions. prioritizing the engagements.
La réponse (B) est fausse. L'examen de la dotation en personnel, de l'ordre Answer (B) is incorrect. By reviewing staffing, prioritization of
de priorité des missions et des dépenses permet d'améliorer le engagements, and expenses, operating benefits can be achieved.
fonctionnement. Answer (C) is correct. The IPPF states that the goals of the
La réponse (C) est juste. Le CRIPP indique que les objectifs de l'audit internal audit activity, as stated in specific operating plans
interne doivent pouvoir être atteints dans le cadre des plans and budgets, should include measurement criteria and
opérationnels et des budgets fixés, et complétés par des critères de targeted dates of accomplishment.
mesure et un calendrier de réalisation. Answer (D) is incorrect. Staffing for each engagement would
La réponse (D) est fausse. Pour chaque mission, la dotation en personnel include this consideration.
prendrait ce facteur en compte.
P1 - 55 -- Deux auditeurs internes sont récemment partis et n'ont pas pu être P1 - 55 -- The internal audit activity has recently experienced the
remplacés tout de suite, pour des raisons budgétaires. Laquelle des departure of two internal auditors who cannot be immediately
méthodes suivantes constitue l'option la moins souhaitable pour la réalisation replaced due to budget constraints. Which of the following is the
efficiente des missions à venir, étant donné cette réduction des ressources ? least desirable option for efficiently completing future engagements,
given this reduction in resources?
A. Répondre aux objectifs des audits au moyen de questionnaires d'auto- A. Using self-assessment questionnaires to address audit
évaluation. objectives.
B. Recourir aux technologies de l'information pour la planification, B. Employing information technology in audit planning,
l'échantillonnage et la documentation des audits. sampling, and documentation.
C. Supprimer les missions de conseil du programme d'audit. C. Eliminating consulting engagements from the engagement work
D. Pourvoir les postes vacants avec du personnel venant de services schedule.
d'exploitation qui ne sont pas en cours d'audit. D. Filling vacancies with personnel from operating departments that
are not being audited.
La réponse (A) est fausse. Un questionnaire d'auto-évaluation constitue un Answer (A) is incorrect. Self-assessment questionnaires are a
moyen d'atteindre avec efficience les objectifs de certains audits internes. means of efficiently addressing the objectives of certain internal
La réponse (B) est fausse. Le recours à la technologie constitue un moyen audits.
approprié de réaliser un audit avec efficience. Answer (B) is incorrect. Use of technology is an appropriate means
La réponse (C) est juste. Le programme d'audit ne doit être allégé qu'en of achieving efficiencies in audit execution.
dernier recours, lorsque toutes les autres solutions viables ont été Answer (C) is correct. The audit schedule should only be
explorées, y compris la demande de ressources supplémentaires. reduced as a last resort once all other viable alternatives have
La réponse (D) est fausse. Le recours à du personnel d'exploitation manifestant been explored, including the request for additional resources.
de l'intérêt pour l'audit interne et disposant d'une expérience de l'entreprise Answer (D) is incorrect. Using operating personnel with internal
constitue un moyen approprié de renforcer les ressources de l'audit interne. audit interest and corporate experience is an appropriate way to
enhance internal audit resources.
Restitution
P1 - 57 -- Lequel des facteurs suivants n'est pas à prendre en compte pour P1 - 57 -- Which of the following factors is not included in
fixer l'ordre des priorités du plan d'audit ? determining the engagement work schedule?
A. Les programmes de travail des missions. A. Engagement work programs.
B. L'efficacité des dispositifs de management des risques et de contrôle. B. The effectiveness of risk management and control processes.
C. Les exigences relatives à la charge de travail. C. Workload requirements.
D. Les aspects relatifs au gouvernement d'entreprise. D. Issues relating to organizational governance.
La réponse (A) est juste. “Le programme de missions doit préciser : a) Answer (A) is correct. Engagement work schedules should
les activités à auditer, b) la date des missions, c) une estimation du include (a) what activities are to be performed; (b) when they
temps nécessaire tenant compte de l'étendue de la mission prévue will be performed; and (c) the estimated time required, taking
ainsi que de la nature et de l'étendue des travaux d'audit réalisés par into account the scope of the engagement work planned and
d'autres intervenants. Les points à prendre en considération pour the nature and extent of related work performed by others.
l'établissement des ordres de priorité sont : a) la date et les résultats de Matters to be considered in establishing engagement work
la précédente mission ; b) une évaluation à jour des risques et de schedule priorities should include (a) the dates and results of
l'efficacité des dispositifs de management des risques et de contrôle ; the last engagement; (b) updated assessments of risks and
c) les demandes émanant de la direction générale, du comité d'audit et effectiveness of risk management and control processes; (c)
d'autres organes de direction ; d) les problématiques actuelles relatives requests by senior management, the audit committee, and the
au gouvernement d'entreprise ; e) les changements importants governing body; (d) current issues relating to organizational
survenus dans l'activité, les tâches opérationnelles, les programmes, governance; (e) major changes in the enterprise's business,
les systèmes et les contrôles ; f) les opportunités de réaliser des operations, programs, systems, and controls; (f)
bénéfices d'exploitation ; g) les modifications intervenues dans l'équipe opportunities to achieve operating benefits; and (g) changes
d'audit interne et ses aptitudes. Le planning des travaux doit être in and capabilities of the staff. The work schedules should be
suffisamment flexible pour faire face à des demandes de missions non sufficiently flexible to cover unanticipated demands on the
prévues” (CRIPP). L'élaboration des programmes de travail a lieu IAA (IPPF). Development of work programs occurs during the
pendant la phase de planification de la mission d'audit. planning phase of an individual engagement.
La réponse (B) est fausse. Les points à prendre en considération pour fixer Answer (B) is incorrect because determining an engagement work
l'ordre des priorités incluent l'évaluation de l'efficacité des dispositifs de schedule includes considering the effectiveness of risk
management des risques et de contrôle. management and control processes.
La réponse (C) est fausse. Les points à prendre en considération pour fixer Answer (C) is incorrect because determining an engagement work
l'ordre des priorités incluent les exigences relatives à la charge de travail. schedule includes considering workload requirements.
La réponse (D) est fausse. Les points à prendre en considération pour fixer Answer (D) is incorrect because determining an engagement work
l'ordre des priorités incluent les aspects relatifs au gouvernement d'entreprise. schedule includes considering issues relating to organizational
governance.
Restitution
P1 - 59 -- Lequel des éléments suivants ne sera probablement pas inclus P1 - 59 -- Which of the following is least likely to be included in the
dans la planification des missions de l’audit interne ? engagement work schedule of the internal audit activity?
A. Être cohérent avec sa charte. A. To be consistent with its charter.
B. Être réalisable. B. To be capable of being accomplished.
C. Inclure une liste d’activités à accomplir. C. To include a list of activities to be performed.
D. Inclure les bases du programme de travail des missions. D. To include the basics of the engagement work program.
La réponse (A) est fausse. La cohérence avec la charte est nécessaire. Answer (A) is incorrect because consistency with the charter is
La réponse (B) est fausse. Les objectifs doivent être réalisables. necessary.
La réponse (C) est fausse. Le programme d’une mission inclut les Answer (B) is incorrect because goals should be capable of being
tâches à accomplir. accomplished.
La réponse (D) est juste. Le responsable de l’audit interne doit établir Answer (C) is incorrect because an engagement work schedule
une planification fondée sur les risques afin de définir les priorités includes the activities to be performed.
cohérentes avec les priorités de l’audit interne (CRIPP). Cette Answer (D) is correct. The chief audit executive should
planification doit être cohérente avec la charte de l’AI et avec les establish risk-based plans to determine the priorities of the
objectifs de l’organisation. De plus, les objectifs de l’AI doivent être internal audit activity (IPPF). These plans should be
réalisables à travers des plans opérationnels et des budgets qui ont été consistent with the IAA's charter and with the goals of the
définis et, dans la mesure du possible, doivent être quantifiables. Ils organization. Moreover, the goals of the IAA should be
doivent être accompagnés de critères de mesure et des dates de capable of being accomplished within specified operating
réalisation. De plus, la planification des missions doit inclure (a) les plans and budgets and, to the extent possible, should be
activités qui doivent être réalisées; (b) quand elles seront réalisées; et measurable. They should be accompanied by measurement
(c) une estimation du temps nécessaire, prenant en considération le criteria and targeted dates of accomplishment. Furthermore,
champ de la mission ainsi que la nature et l’étendue de tous les travaux engagement work schedules should include (a) what
effectués par d’autres. (CRIPP). Les éléments fondamentaux d’un activities are to be performed; (b) when they will be
programme pour une mission spécifique ne sont pas inclus dans les performed; and (c) the estimated time required, taking into
plans à long terme de l’AI. Un programme de travail documente les account the scope of the engagement work planned and the
procédures relatives à la mission et choisies avant de conduire la nature and extent of related work performed by others (IPPF).
mission. Cependant, il est normalement préparé après que les The basics of an engagement program for a specific
informations de base et de contexte ont été recueillies et une étude engagement are not included in the long-range plans of the
préliminaire réalisée. IAA. A work program documents the engagement procedures
selected in advance of performing the engagement. However,
it is normally prepared after background information has been
gathered and a survey has been conducted.
Restitution
Restitution
Restitution
Restitution
Restitution
La réponse (A) est fausse, car élaborer des évaluations et rédiger Answer (A) is incorrect because developing assessments and
des rapports sur le processus de gestion des risques de reports on the organization's risk management processes is not
l'organisation fait non seulement partie du rôle de l'audit interne, only an internal auditing role, but it is normally also a high audit
mais figure normalement en haut de ses priorités. priority.
La réponse (B) est juste. Jouer un rôle plus proactif en Answer (B) is correct. A more proactive role in the initial
participant à la mise en place initiale d'un processus de establishment of a risk management process supplements
management des risques complète la mission classique traditional assurance activities with a consultative approach
d'assurance. Les auditeurs internes assument alors un rôle to improving fundamental processes. However, a proactive
de conseil en vue de l'amélioration des processus role in developing a risk management process is not the
fondamentaux. Il convient de différencier le rôle proactif de same as an "ownership of risks" role, which is a role of
l'auditeur dans la mise en place d'un dispositif de management (IPPF). The internal auditor cannot assume this
management des risques, du rôle de « responsable des role, or any other management, board, or audit committee
risques » qui incombe à la direction (CRIPP). L'auditeur role in the risk management process, without impairing
interne ne peut assumer ce rôle, ni n'importe quel autre rôle independence. Boards and audit committees have an
de la direction, du conseil ou du comité d'audit dans le oversight role to determine that adequate and effective
processus de management des risques, sans porter atteinte à processes are in place, and managers have the responsibility
son indépendance. Il incombe au Conseil et au comité d'audit for the management of the risks identified, and for ensuring
de veiller à ce que des processus de management des that sound risk management processes are in place and
risques appropriés, suffisants et efficaces soient en place et functioning (IPPF). By managing the identified risks, the
la gestion des risques est une responsabilité majeure du internal auditor impairs his/her independence by assuming
management qui doit s'assurer, pour atteindre ses objectifs, management's role.
de la mise en place et du bon fonctionnement des processus Answer (C) is incorrect because internal auditors should assist
de management des risques (CRIPP). En gérant les risques both management and the audit committee by examining,
identifiés, l'auditeur interne porte atteinte à son indépendance evaluating, reporting, and recommending improvements on the
car il assume un rôle qui incombe à la direction. adequacy and effectiveness of management's risk processes.
La réponse (C) est fausse, car « le rôle des auditeurs internes Answer (D) is incorrect because internal auditors acting in a
consiste à assister et le management et le comité d'audit, en consulting role may assist the organization in identifying,
examinant et en évaluant les processus de management des evaluating, and implementing risk management methodologies
risques mis en oeuvre par le management, en vérifiant qu'ils sont and controls to address those risks (IPPF).
suffisants et efficaces, puis en émettant des rapports et des
recommandations en vue de leur amélioration ».
La réponse (D) est fausse, car « les auditeurs internes peuvent,
dans le cadre d'une mission de conseil, aider l'organisation à
identifier, évaluer et mettre en place un dispositif de management
des risques et des contrôles permettant de maîtriser ces risques »
(CRIPP).
Restitution
A. La définition des objectifs et du champ d'intervention. A. Establishing engagement objectives and scope of work.
B. L'obtention d'informations sur le contexte des activités B. Obtaining background information about the activities to be
examinées. reviewed.
C. La collecte d'informations suffisantes pour permettre d'atteindre C. Identifying sufficient information to achieve engagement
les objectifs de la mission. objectives.
D. La détermination des moyens, du moment et des personnes à D. Determining how, when, and to whom the engagement results
qui les résultats de la mission seront communiqués. will be communicated.
La réponse (A) est fausse. La planification inclut la définition des Answer (A) is incorrect because the planning process includes
objectifs et du champ d'application. establishing engagement objectives and scope of work.
La réponse (B) est fausse. La planification inclut l'obtention Answer (B) is incorrect because the planning process includes
d'informations sur le contexte. obtaining background information.
La réponse (C) est juste. La planification doit notamment Answer (C) is correct. Planning should include establishing
définir les objectifs et le champ d'application de la mission, engagement objectives and scope of work, obtaining
obtenir des informations sur le contexte des activités background information about the activities to be reviewed,
examinées, déterminer les ressources appropriées pour determining the resources necessary to perform the
atteindre les objectifs de la mission et informer les niveaux engagement, and informing those in management who need
hiérarchiques qui doivent être au courant de la mission. Elle to know about the engagement. It also includes performing,
inclut également la réalisation, si nécessaire, d'un examen as appropriate, a survey to become familiar with activities,
préliminaire sur place destiné à se familiariser avec les risks, and controls; to identify areas for engagement
activités, les risques et les contrôles, à identifier les emphasis; and to invite comments and suggestions from
domaines où la mission devra être approfondie et à inviter les engagement clients. Furthermore, planning extends to
clients de la mission à formuler leurs commentaires et developing work programs; determining how, when, and to
suggestions. De plus, la planification recouvre l'élaboration whom engagement results will be communicated; and
de programmes de travail, la détermination de comment, obtaining approval of the engagement work program.
quand et à qui les résultats de l'audit seront communiqués. Identifying sufficient information to achieve engagement
La planification inclut également l'approbation du programme objectives is done during field work, not planning.
de travail de la mission. La collecte d'informations suffisantes Answer (D) is incorrect because the planning process includes
pour permettre d'atteindre les objectifs de la mission determining how, when, and to whom the engagement results will
s'effectue durant le travail sur place et non pendant la be communicated.
planification.
La réponse (D) est fausse. La planification inclut la détermination
des moyens, du moment et des personnes à qui les résultats de la
mission seront communiqués.
Restitution
La réponse (A) est fausse. Ne pas réaliser la mission est Answer (A) is incorrect because not performing the engagement
inacceptable, particulièrement dans un domaine à haut risque. is unacceptable, especially for a high-risk area.
La réponse (B) est fausse. Les missions doivent être correctement Answer (B) is incorrect because engagements should be properly
supervisées. Le service d’audit interne n’a personne pour prendre supervised. The IAA has no one to provide this supervision.
en charge cette supervision. Answer (C) is correct. In determining the resources needed
La réponse (C) est juste. En identifiant les ressources to perform the engagement, the CAE should consider the
nécessaires à la réalisation de la mission, le RAI doit tenir knowledge, skills, and other competencies of the internal
compte des savoirs, connaissances, et autres compétences auditing staff in selecting internal auditors for the
du personnel de l’audit interne en choisissant les auditeurs engagement. The CAE should consider the use of external
internes. Le RAI doit envisager le recours à des ressources resources when additional knowledge, skills, and other
externes lorsque des savoirs, connaissances, et autres competencies are needed (IPPF).
compétences du personnel sont nécessaires (CRIPP). Answer (D) is incorrect because limiting the scope of the
La réponse (D) est fausse. Limiter le périmètre de la mission ne engagement should be done only when the requisite skills are not
devrait être fait que lorsque les connaissances requises ne sont available even from external resources. If the scope is limited,
pas non plus disponibles en externe. Si le périmètre de la mission management should be informed of the constraint in an interim
est limité, la direction devrait en être informée dans un rapport report.
intermédiaire.
P1- 67 -- Lors de la planification d'une mission d'assurance, un P1- 67 -- In planning an assurance engagement, a survey could
examen préliminaire peut permettre la réalisation des actions assist with all of the following, except
suivantes, sauf
A. Obtenir les commentaires et suggestions des clients de la A. Obtaining engagement client comments and suggestions on
mission sur les problèmes de contrôle. control problems.
B. Obtenir des informations préliminaires sur les contrôles. B. Obtaining preliminary information on controls.
C. Identifier les secteurs importants nécessitant une analyse plus C. Identifying areas for engagement emphasis.
approfondie. D. Evaluating the adequacy and effectiveness of controls.
D. Évaluer le caractère adéquat et l'efficacité des contrôles.
La réponse (A) est fausse. Un examen préliminaire peut permettre Answer (A) is incorrect because a survey could assist with
d'obtenir les commentaires et suggestions des clients de la obtaining client comments and suggestions on control problems.
mission sur les problèmes de contrôle. Answer (B) is incorrect because a survey could assist with
La réponse (B) est fausse. Un examen préliminaire peut permettre obtaining preliminary information on controls.
d'obtenir des informations préliminaires sur les contrôles. Answer (C) is incorrect because a survey could assist with
La réponse (C) est fausse. Un examen préliminaire peut permettre identifying areas for engagement emphasis.
d'identifier les domaines importants nécessitant une analyse plus Answer (D) is correct. A survey is a process for gathering
approfondie. information, without detailed verification, on the activity
La réponse (D) est juste. « Un examen préliminaire est un being examined. A survey may involve discussions with the
procédé permettant de rassembler des informations sur client, documenting key control activities, and identifying
l'activité étudiée, sans vérification détaillée. Un examen significant engagement issues (IPPF). A survey does not help
préliminaire peut entraîner l'utilisation des procédures in evaluating the adequacy and effectiveness of controls
suivantes : les discussions avec le client de la mission, la except to the extent the internal auditor gains familiarity with
documentation des activités de contrôle clé » et the controls. Evaluation requires testing.
l'identification des problèmes importants (CRIPP). Un examen
préliminaire ne permet pas d'évaluer le caractère adéquat et
l'efficacité des contrôles, sauf dans la mesure où l'auditeur
interne se familiarise avec les contrôles. L'évaluation
nécessite la réalisation de tests.
Restitution
Restitution
La réponse (A) est fausse. L'auditeur interne doit enquêter sur la Answer (A) is incorrect because the internal auditor needs to
cause de l'augmentation des coûts et sur la qualité des prestations investigate the cause for the increase in costs and the accuracy
de l'organisme de traitement des demandes. L'auditeur interne with which the claims processor is handling claims. The internal
doit, dans le cadre de son activité ordinaire, interroger la direction auditor should routinely interview engagement client management
du client de la mission mais aussi envisager de faire appel à un and should also consider engaging an actuarial consultant.
actuaire. Answer (B) is incorrect because the internal auditor needs to
La réponse (B) est fausse. L'auditeur interne doit enquêter sur la investigate the cause for the increase in costs and the accuracy
cause de l'augmentation des coûts et sur la qualité des prestations with which the claims processor is handling claims. The internal
de l'organisme de traitement des demandes. L'auditeur interne auditor should routinely interview engagement client management
doit, dans le cadre de son activité ordinaire, interroger la direction and should also consider engaging an actuarial consultant.
du client de la mission mais aussi envisager de faire appel à un Answer (C) is incorrect because the internal auditor needs to
actuaire. investigate the cause for the increase in costs and the accuracy
La réponse (C) est fausse. L'auditeur interne doit enquêter sur la with which the claims processor is handling claims. The internal
cause de l'augmentation des coûts et sur la qualité des prestations auditor should routinely interview engagement client management
de l'organisme de traitement des demandes. L'auditeur interne and should also consider engaging an actuarial consultant.
doit, dans le cadre de son activité ordinaire, interroger la direction Answer (D) is correct. One step in planning the engagement
du client de la mission mais aussi envisager de faire appel à un is to define engagement objectives (intended engagement
actuaire. accomplishments) and procedures (means of achieving the
La réponse (D) est juste. L'une des étapes de la panification objectives). Taken together, the objectives and procedures
de la mission consiste à définir les objectifs (réalisations define the scope of the internal auditor's work (PA 2210-1).
attendues de la mission) et les procédures (moyens pour Objectives and procedures are documented in the
atteindre les objectifs). « L'ensemble des objectifs et des engagement work program. Before the work program can be
procédures de la mission définissent le champ d'intervention drafted, however, a preliminary survey is conducted. This
de l'auditeur interne » (CRIPP). Les objectifs et les procédures survey includes, among other steps, discussions with the
sont consignés dans le programme de travail de la mission. engagement client to increase the internal auditor's
Toutefois, avant que le programme de travail ne soit élaboré, familiarity with the activities to be reviewed (IPPF).
un examen préliminaire est mené. Celui-ci comprend, entre Furthermore, if the internal auditing staff does not have the
autres, les discussions avec le client de la mission destinées requisite knowledge, skills, and other competencies to
à permettre à l'auditeur interne de se familiariser avec les perform all or part of the engagement, the CAE should obtain
activités à examiner (CRIPP). En outre, « le responsable de competent advice and assistance (IPPF). However, the
l'audit interne doit obtenir l'avis et l'assistance de personnes outside claims processing function is an integral part of the
qualifiées si les auditeurs internes ne possèdent pas les organization's internal control. Thus, the scope of the
connaissances, le savoir-faire et les autres compétences internal auditor's work should extend to the adequacy and
nécessaires pour s'acquitter de tout ou partie de leur mission effectiveness of internal control over claims processing.
» (CRIPP). Cependant, le prestataire extérieur chargé du
traitement des demandes fait aussi partie du dispositif de
contrôle interne de l'organisation. Le travail de l'auditeur
interne doit donc également porter sur le caractère adéquat et
l'efficacité du contrôle interne en matière de traitement des
demandes.
Restitution
P1- 71 -- Lors d’une réunion avec les chefs de missions, le P1- 71 -- At a meeting with engagement managers, the chief audit
directeur d’audit attribue le calendrier des missions d’audit pour executive is allocating the engagement work schedule for next
l’année à venir. Laquelle des méthodes suivantes garantira que year's plan. Which of the following methods will ensure that each
chaque responsable recevra une part équitable aussi bien du manager receives an appropriate share of both the work schedule
calendrier de travail que des ressources allouées ? and internal audit activity resources?
A. Le travail est attribué à chaque responsable à partir d’une A. Work is assigned to each manager based on risk and skill
analyse de risque et de compétence. analysis.
B. Chacun des responsables sélectionne les missions qu’il B. Each of the managers selects the individual assignments
souhaite réaliser, en fonction des départements qu’il affectionne et desired, based on preferences for the area and the management
des managers qui les dirigent. personnel involved.
C. Chaque responsable choisit ses missions en fonction du C. Each manager chooses assignment preferences based on the
nombre total d’heures / homme à sa disposition. total staff hours that are currently available to each manager.
D. La liste complète des missions planifiées est publiée à D. The full list of scheduled engagements is published for the
l’attention de l’ensemble du personnel et leur attribution est faite staff, and work assignments are made based on career interests
en fonction des souhaits de carrière et des voyages que les and travel requirements.
auditeurs souhaitent effectuer.
La réponse (A) est correcte. Les missions doivent être Answer (A) is correct. Engagements should be performed
conduites avec compétence et conscience professionnelle. with proficiency and due professional care. Thus,
Ainsi, la conscience professionnelle doit être à la hauteur de professional care should be commensurate with the
la complexité de la mission, et l’audit interne doit s’assurer complexities of the engagement, and the IAA should assure
que l’équipe d’auditeurs désignée possède collectivement les that the technical proficiency and educational background of
connaissances théoriques et les compétences techniques the personnel assigned are appropriate. A skill analysis of
pour mener à bien la mission. Une analyse de compétences tasks to be performed is therefore necessary. Furthermore,
pour accomplir les tâches à réaliser est ainsi nécessaire. Par matters to be considered in establishing engagement work
ailleurs, l’élaboration du plan d’audit doit tenir compte (1) schedule priorities include (1) updated assessments of risks
d’une évaluation actualisée des risques et de l’efficacité des and effectiveness of risk management and control processes
processus de gestion des risques et de contrôle et (2) des and (2) changes to and capabilities of the staff.
changements au niveau des compétences des salariés. Answer (B) is incorrect because choice based on personal
La réponse (C) est fausse. La sélection basée sur le choix preference does not ensure the exercise of due professional care.
personnel ne garantit pas l’application de la conscience Answer (C) is incorrect because available staff hours do not
professionnelle. correlate with risk or the composite skills necessary for individual
La réponse (C) est fausse. Le nombre d’heures disponible au assignments.
niveau de l’équipe d’audit ne peut être rapproché du risque ou des Answer (D) is incorrect because although career interests and
diverses compétences nécessaires aux missions individuelles. travel requirements are considerations for staffing engagements,
La réponse (D) est incorrecte car bien que les souhaits de carrière these factors do not constitute an objective basis for making
et de voyage sont des considérations à prendre en compte, ces assignments.
facteurs ne constituent pas une base objective d’attribution de
mission.
Restitution
Restitution
P1 - 74 -- Lors d'une mission d'assurance portant sur des opérations de P1 - 74 -- In an assurance engagement of treasury operations, an
trésorerie, un auditeur interne doit examiner tous les points suivants à internal auditor is required to consider all of the following issues
l'exception duquel ? except:
A. Le comité d'audit demande qu'on lui apporte l'assurance que le service de A. The audit committee has requested assurance on the treasury
la trésorerie respecte les nouvelles règles d'utilisation des instruments department's compliance with a new policy on use of financial
financiers. instruments.
B. La direction de la trésorerie n'a mis en place aucune mesure de B. Treasury management has not instituted any risk management
gestion du risque. policies.
C. En raison de la récente cession d'une division, le montant des liquidités et C. Due to the recent sale of a division, the amount of cash and
des titres négociables gérés par le service de la trésorerie s'est accru de 350 %. marketable securities managed by the treasury department has
D. Les auditeurs externes ont indiqué qu'ils avaient eu des difficultés à obtenir increased by 350 percent.
des confirmations de la part de tiers. D. The external auditors have indicated some difficulties in
obtaining account confirmations.
La réponse (A) est fausse. Selon le CRIPP, l'auditeur doit prendre en Answer (A) is incorrect. The IPPF states that the auditor should
considération l'étendue du travail nécessaire pour atteindre les objectifs de la consider the extent of work needed to achieve the engagement's
mission. Il s'agit d'un objectif de mission spécifique. objectives. This is a specific engagement objective.
La réponse (B) est fausse. Selon le CRIPP, l'auditeur doit prendre en Answer (B) is incorrect. The IPPF states that the auditor should
considération la pertinence et l'efficacité des processus de management des consider the adequacy and effectiveness of risk management
risques. processes.
La réponse (C) est fausse. Selon le CRIPP, l'auditeur doit prendre en Answer (C) is incorrect. The IPPF states that the auditor should
considération la complexité relative, la matérialité ou le caractère significatif consider significance and materiality of matters to which assurance
des domaines auxquels sont appliquées les procédures propres aux procedures are applied. This is a significant increase.
missions d'assurance. L'augmentation est ici significative. Answer (D) is correct. This is the responsibility of the external
La réponse (D) est juste. Ce point relève de la responsabilité des auditors and should not change what should be considered
auditeurs externes et ne doit pas changer ce dont doit tenir compte by the internal auditor.
l'auditeur interne.
Restitution
La réponse (A) est fausse. Cet objectif relève d'une mission de conformité et Answer (A) is incorrect because it states an objective of a
ne concerne pas l'efficience. compliance engagement, not one of efficiency.
La réponse (B) est juste. Les auditeurs internes doivent déterminer si Answer (B) is correct. Internal auditors should appraise the
les ressources sont employées de manière rentable et efficiente. economy and efficiency with which resources are employed.
Confier à des salariés des tâches qui ne correspondent pas à leurs Assignment of employees to tasks not commensurate with
compétences peut entraîner des coûts de main-d'oeuvre excessifs their skills may result in excess labor costs (when more
(lorsque des travailleurs plus qualifiés et mieux rémunérés effectuent skilled and more highly paid workers perform jobs for which
des tâches pour lesquelles ils sont trop qualifiés) ou des performances they are overqualified) or in poor performance (when
médiocres (lorsqu'une main-d'oeuvre sous-qualifiée est employée). underqualified labor is used).
La réponse (C) est fausse. Ce facteur concerne l'efficacité et non l'efficience. Answer (C) is incorrect because it concerns effectiveness, not
La réponse (D) est fausse. Ce facteur ne concerne pas l'efficience. efficiency.
Answer (D) is incorrect because it is irrelevant to efficiency.
P1 - 76 -- Laquelle des caractéristiques suivantes fera le plus P1 - 76 -- Which of the following is most likely to be an element of
vraisemblablement partie d'un programme efficace de conformité à la an effective regulatory compliance program?
réglementation ?
A. The internal audit activity should be assigned overall
A. L'audit interne doit être chargé de superviser le programme. responsibility for overseeing the program.
B. Le programme est communiqué aux salariés une seule fois sous un B. The program is communicated to employees in a video format
format vidéo. on a one-time basis.
C. L'organisation doit exploiter des systèmes de suivi et d'audit conçus pour C. The organization should use monitoring and auditing systems
détecter les pratiques délictueuses. reasonably designed to detect criminal conduct.
D. L'organisation doit réunir autant d'informations que possible lorsqu'elle D. The organization should obtain as much information as possible
procède à la vérification des antécédents des salariés. when performing background checks on employees.
La réponse (A) est fausse. Il convient de désigner au sein de l'organisation un Answer (A) is incorrect because specific individual(s) within high-
ou plusieurs collaborateurs de haut niveau chargés de veiller au respect des level personnel of the organization should be assigned overall
normes et des procédures. L'expression “collaborateurs de haut niveau” vise responsibility to oversee regulatory compliance with standards and
les personnes qui jouent un rôle important dans l'organisation ou dans son procedures. High-level personnel of the organization means
processus de décision. individuals who have substantial control of the organization or who
La réponse (B) est fausse. L'efficacité d'un programme de conformité have a substantial role in the making of policy within the
dépendra du mode de présentation adopté. En règle générale, la organization.
communication de type interactif est plus efficace qu'un exposé formel. De Answer (B) is incorrect because the effectiveness of a compliance
même, les programmes présentés par un animateur fonctionnent program will depend upon the ways in which it is communicated to
généralement mieux que les programmes entièrement présentés sous forme employees. Generally, an interactive format works better than a
de vidéo ou de jeu. L'organisation de sessions périodiques est plus efficace lecture. Programs communicated in person tend to work better
qu'une présentation unique. than programs communicated entirely through video or game
La réponse (C) est juste. L'organisation doit prendre des mesures formats. Programs that are periodically repeated work better than
appropriées afin d'assurer le respect de ses normes. À titre d'exemple, one-time presentations.
ces mesures incluent l'utilisation de systèmes de suivi et d'audit Answer (C) is correct. The organization should take
permettant de détecter les infractions commises par les salariés et reasonable steps to achieve compliance with its standards,
agents, ainsi que la mise en place et la présentation aux salariés et e.g., by using monitoring and auditing systems reasonably
agents de l'organisation, d'un système grâce auquel ils peuvent designed to detect criminal conduct by its employees and
indiquer, sans crainte de représailles, les infractions commises par other agents and by having in place and publicizing a
autrui au sein de l'organisation. reporting system whereby employees and other agents could
La réponse (D) est fausse. L'organisation doit veiller à ne pas déléguer de report criminal conduct by others within the organization
pouvoirs discrétionnaires importants à des personnes dont elle sait, de par sa without fear of retribution.
surveillance, ou devrait savoir, qu'elles sont susceptibles de commettre des Answer (D) is incorrect because the organization should use due
actes illicites. En revanche, toutes les précautions doivent être prises pour care not to delegate substantial discretionary authority to
s'assurer que l'organisation ne viole pas les lois protégeant la vie privée des individuals the organization knows, or should know through the
salariés et des candidats. exercise of due diligence, have a propensity to engage in illegal
activities. However, care should be taken to ensure that the
organization does not infringe upon employees' and applicants'
privacy rights under applicable laws.
Restitution
P1 - 77 -- (Voir énoncé 5) Le comité d’audit a exprimé son inquiétude sur le P1 - 77 -- (Refers to Fact Pattern #5) The audit committee has
fait que l’institution financière s’est engagée sur des crédits plus risqués afin expressed concern that the financial institution has been taking on
de réaliser des objectifs de rentabilité à court terme. Laquelle des procédures higher-risk loans in pursuit of short-term profit goals. Which of the
de mission suivantes fournit le moins d’informations pour répondre à cette following engagement procedures provides the least amount of
inquiétude ? information to address this concern?
A. Effectuer une revue analytique des intérêts créditeurs en tant que A. Perform an analytical review of interest income as a percentage
pourcentage de la valeur du portefeuille d’investissements comparé à un of the investment portfolio in comparison with a group of peer
groupe d’institutions financières comparables. financial institutions.
B. Prendre, au hasard, un échantillon de prêts accordés pendant la période B. Take a random sample of loans made during the period and
et comparer le niveau de risque des prêts avec celui d’un échantillon de compare the riskiness of the loans with that of a random sample of
prêts, pris au hasard, accordés il y a deux ans. loans made 2 years ago.
C. Effectuer une revue analytique qui implique la création d’un tableau C. Perform an analytical review that involves developing a chart to
comparatif des intérêts créditeurs tracé sur les dix dernières années. compare interest income plotted over the past 10 years.
D. Effectuer une analyse de régression multiple par série chronologique des D. Develop a multiple-regression time-series analysis of income
intérêts créditeurs sur les dernières cinq années en incluant des facteurs tels over the past 5 years including such factors as interest rate in the
que les taux d’intérêts dans l’économie, l’importance du portefeuille des economy, size of loan portfolio, and dollar amount of new loans
crédits, et la valeur en dollars des nouveaux crédits chaque année. each year.
La réponse (A) est fausse. Les crédits plus risqués doivent générer des intérêts Answer (A) is incorrect because higher-risk loans should generate
plus élevés à court terme comparés à ceux d’autres institutions similaires. Les higher short-term interest income compared with that earned by
crédits qui sont plus risqués fournissent une rémunération plus élevée. comparable institutions. Higher-risk loans have higher yields.
La réponse (B) est fausse. Une comparaison historique du risque des prêts Answer (B) is incorrect because a historical comparison of loan risk
encourus par l’institution répond à l’objectif de la mission. for the institution addresses the engagement objective.
La réponse (C) est juste. Le fait de tracer les fluctuations des intérêts Answer (C) is correct. Plotting the changes in interest income
créditeurs sur les dix dernières années est l’approche la moins utile. over the past 10 years is the least useful procedure. It does
Cela ne prend pas en compte d’autres facteurs importants, tels que not consider other important factors, such as size of the
l’importance du portefeuille, les changements des taux d’intérêts, le portfolio, changes in interest rates, the development of new
développement de nouveaux instruments financiers, le taux d’inflation, financial instruments, the level of inflation, and government
et la réglementation gouvernementale. regulation.
La réponse (D) est fausse. La régression multiple explique la fluctuation d’une Answer (D) is incorrect because multiple regression explains the
variable dépendante (les intérêts créditeurs) imputable à deux variables change in a dependent variable (interest income) attributable to two
indépendantes ou plus. Il permet donc à l’auditeur d’estimer quelle proportion de la or more independent variables. Thus, it allows the internal auditor
fluctuation pourrait être attribuable au niveau de risque des crédits accordés. to estimate how much of the change might be due to a change in
the riskiness of the loans.
P1 - 78 -- Lequel des concepts suivants ne fait pas partie des concepts P1 - 78 -- Which of the following is not an interrelated concept that
interdépendants fondamentaux pour le champ de l'audit interne et le travail is fundamental to the field of internal auditing and the work of
des auditeurs internes ? internal auditors?
A. La gouvernance. A. Governance.
B. L'importance relative. B. Materiality.
C. Le risque. C. Risk.
D. Le contrôle. D. Control.
La réponse (A) est fausse. Il fait partie des concepts interdépendants Answer (A) is incorrect because it is an interrelated concept that is
fondamentaux pour le champ de l'audit interne et le travail des auditeurs internes. fundamental to the field of internal auditing and the work on internal
La réponse (B) est juste. La gouvernance, le risque et le contrôle sont auditors.
des concepts interdépendants fondamentaux pour le champ de l'audit Answer (B) is correct. Governance, risk, and control are
interne et le travail des auditeurs internes. L'importance relative est un interrelated concepts that are fundamental to the field of internal
concept lié à l'audit externe. auditing and the work of internal auditors. Materiality is a concept
La réponse (C) est fausse. Il fait partie des concepts interdépendants related to external auditing.
fondamentaux pour le champ de l'audit interne et le travail des auditeurs Answer (C) is incorrect because it is an interrelated concept that is
internes. fundamental to the field of internal auditing and the work on internal
La réponse (D) est fausse. Il fait partie des concepts interdépendants auditors.
fondamentaux pour le champ de l'audit interne et le travail des auditeurs internes. Answer (D) is incorrect because it is an interrelated concept that is
fundamental to the field of internal auditing and the work on internal
auditors.
Restitution
P1 - 80 -- Des informations sur les billets d'avion, notamment le tarif, la P1 - 80 -- Specific airline ticket information, including fare, class,
classe, la date d'achat et les options tarifaires les plus économiques purchase date, and lowest available fare options, as prescribed in
disponibles, exigées par la politique de l'organisation relative aux the organization's travel policy, is obtained and reported to
déplacements, sont collectées et communiquées à la direction du department management when employees purchase airline tickets
département lorsque les salariés achètent des billets d'avion auprès de from the organization's authorized travel agency. Such a report
l'agence de voyage agréée par l'organisation. De tels éléments donnent des provides information for
informations :
A. Sur la qualité de ce qui est fait par rapport à la politique de l'organisation A. Quality of performance in relation to the organization's travel
sur les déplacements. policy.
B. Pour identifier les coûts nécessaires au traitement des données relatives B. Identifying costs necessary to process employee business
aux frais de déplacement des salariés. expense report data.
C. Pour comparer les dépenses budgétées et réelles au niveau du service C. Departmental budget-to-actual comparisons.
concerné. D. Supporting employer's business expense deductions.
D. Pour justifier les frais de déplacement professionnels de l'employeur.
La réponse (A) est juste. Le retour d'information fait partie du cycle de Answer (A) is correct. Feedback is a part of the internal
contrôle interne. Il forme une base de comparaison entre les control cycle. It provides a basis for comparing actual
performances réelles (achats de billets en fonction des options performance (purchases of tickets given the available
disponibles) et les normes (la politique de l'organisation). options) with standards (organizational policy).
La réponse (B) est fausse. Ces informations sont préliminaires. Les salariés Answer (B) is incorrect because this ticket information is
peuvent modifier leurs billets et leurs itinéraires avant leur déplacement. preliminary; employees may change tickets and routings prior to
La réponse (C) est fausse. La comparaison des dépenses budgétées et their trip.
réelles au niveau du service ne reflète pas forcément les coûts réels. Answer (C) is incorrect because departmental budget-to-actual
La réponse (D) est fausse. Les justificatifs pour les frais de déplacement ne comparisons do not necessarily reflect the actual costs ultimately
reflètent pas forcément les coûts réels. incurred.
Answer (D) is incorrect because supporting expense deductions
may not necessarily reflect actual costs.
Restitution
P1 - 82 -- Lequel des énoncés suivants décrit le mieux la finalité de l'examen, P1 - 82 -- Which of the following best describes an internal auditor's
par un auditeur interne, des processus de gestion du risque, de contrôle et de purpose in reviewing the organization's existing risk management,
gouvernance de l'organisation ? control, and governance processes?
A. Aider à déterminer la nature, la durée et l'étendue des contrôles A. To help determine the nature, timing, and extent of tests
nécessaires à la réalisation des objectifs de la mission. necessary to achieve engagement objectives.
B. Vérifier que les déficiences du système de contrôle interne ont été corrigées. B. To ensure that weaknesses in the internal control system are
C. Apporter une assurance raisonnable que ces processus permettront la corrected.
réalisation efficiente et économique des objectifs et buts de l'organisation. C. To provide reasonable assurance that the processes will enable
D. Déterminer si ces processus permettent des écritures comptables the organization's objectives and goals to be met efficiently and
correctes et l'élaboration d'états financiers justes. economically.
D. To determine whether the processes ensure that the accounting
records are correct and that financial statements are fairly stated.
La réponse (A) est fausse. C'est la finalité de la planification de l'audit. Answer (A) is incorrect. This is a purpose of audit planning.
La réponse (B) est fausse. C'est à la direction, et non à l'auditeur interne, de Answer (B) is incorrect. Correcting control weaknesses is a
corriger les déficiences du contrôle. function of management, not of the internal auditor.
La réponse (C) est juste. C'est la finalité énoncée dans le CRIPP. Answer (C) is correct. This is the purpose stated in IPPF.
La réponse (D) est fausse. C'est un objectif fondamental du point de vue de Answer (D) is incorrect. This is a basic objective from a financial
l'audit et de la certification des comptes, mais il ne décrit pas la finalité globale accounting and auditing perspective, but is not broad enough to
de l'examen mené par l'auditeur interne. cover the internal auditor's entire purpose for review.
Restitution
La réponse (A) est fausse. Le directeur général est un membre de la Answer (A) is incorrect. The chief executive officer (CEO) is a
direction. D'autres membres de la direction peuvent recevoir la version member of senior management. Other members of senior
définitive du rapport qui a été révisé et approuvé par le conseiller juridique. management may receive a final report that has been reviewed
La réponse (B) est fausse. Les auditeurs externes ne doivent pas être and approved by legal counsel.
contactés. Ils peuvent recevoir la version définitive du rapport qui a été révisé Answer (B) is incorrect. External auditors should not be contacted.
et approuvé par le conseiller juridique. External auditors may be given a final report that has been
La réponse (C) est juste. Il convient de présenter un projet du rapport reviewed and approved by legal counsel.
faisant état de la fraude ou du conflit d'intérêts au président du comité Answer (C) is correct. A draft of the proposed report on fraud
d'audit, étant donné la position du DG au sein de l'entreprise. or conflict-of-interest situations should be submitted to the
La réponse (D) est fausse. Il convient de disposer des pièces justificatives chairman of the audit committee as a next step in light of the
avant d'informer le comité d'audit ou le conseil. CEO's position in the company.
Answer (D) is incorrect. Supporting documentation would be
necessary before informing the audit committee or the board.
P1 - 84 -- La direction et le conseil d'administration sont chargés de mettre en P1 - 84 -- Management and the board of directors are responsible for
place des mesures correctives formulées à la suite des observations et des following up on observations and recommendations made by the
recommandations des auditeurs externes. Quel est le rôle de l'audit interne external auditors. What role, if any, should the internal audit activity
dans ce processus, le cas échéant ? have in this process?
A. L'audit interne ne joue aucun rôle dans ce processus pour des impératifs A. The internal audit activity should have no role in this process in
d'indépendance. order to ensure independence.
B. L'audit interne ne doit y prendre part que si son responsable dispose de B. The internal audit activity should only become involved if the
preuves suffisantes d'une absence de mesures correctives. chief audit executive has sufficient evidence that the follow-up is
C. L'audit interne doit instaurer un processus de suivi afin de vérifier le bien- not occurring.
fondé et l'efficacité des mesures correctives mises en place par la direction. C. The internal audit activity should establish a monitoring process
D. L'audit interne ne doit y prendre part uniquement si elle y est to review the adequacy and effectiveness of management's follow-
expressément conviée par la direction ou le conseil d'administration. up actions.
D. The internal audit activity should become involved only if
specifically requested by management or the board of directors.
Restitution
P1 - 86 -- Lequel des processus suivants constitue un contrôle opérationnel P1 - 86 -- Which of the following is an operating control relating to
de la fonction de gestion exercée par la direction ? management's directing function?
A. Informer suffisamment à l'avance le service des achats des futurs besoins A. Informing purchasing personnel of the future need for long-lead-
en produits nécessitant un long délai d'approvisionnement. time products in ample time.
B. Présenter au service des achats des comptes rendus élaborés B. Supplying buyers with timely, accurate, and useful reports on
rapidement, exacts et utiles sur les produits reçus, acceptés ou rejetés. products received, accepted, or rejected.
C. Définir des procédures formelles pour sélectionner les fournisseurs potentiels. C. Prescribing formal procedures for selecting potential suppliers.
D. Fixer des objectifs mesurables pour ce service. D. Establishing measurable goals for the department.
La réponse (A) est juste. La gestion est la fonction consistant à motiver Answer (A) is correct. Directing is the function of motivating
les ressources humaines d'une organisation afin qu'elles contribuent people in an organization to contribute effectively and
avec efficacité et efficience à la réalisation des objectifs et buts de efficiently to the achievement of enterprise objectives and
l'entreprise (le CRIPP donne une définition plus large de la gestion). goals. (IPPF provides a broader definition of directing.)
Avertir rapidement le personnel des changements au niveau de leurs Providing adequate notice to employees of changes in their
activités constitue une facette d'un bon leadership. duties is a facet of proper leadership.
La réponse (B) est fausse. Le retour d'information rapide est une Answer (B) is incorrect because providing timely feedback is a control
fonction de contrôle. function.
La réponse (C) est fausse. La définition de procédures formelles pour Answer (C) is incorrect because prescribing formal procedures for
sélectionner des fournisseurs potentiels est une fonction du processus de selecting potential suppliers is a function of the planning process.
planification. Answer (D) is incorrect because establishing measurable goals for
La réponse (D) est fausse. La fixation d'objectifs mesurables pour le service the department is a function of the planning process.
concerné est une fonction du processus de planification.
Restitution
P1 - 88 -- Le champ d'une mission d'assurance doit tenir compte : P1 - 88 -- The scope of an internal auditing assurance engagement
should include consideration of
A. Uniquement des systèmes et documents sous le contrôle du client de la A. Only those systems and records under the control of the
mission. engagement client.
B. Des biens pertinents sous le contrôle de tiers. B. Relevant physical properties under third-party control.
C. Des observations, conclusions et recommandations de la mission. C. Engagement observations, conclusions, and recommendations.
D. Des communications finales de la mission. D. Final engagement communications.
La réponse (A) est fausse. Les systèmes, les documents, le personnel et les Answer (A) is incorrect because systems, records, personnel, and
biens physiques sous le contrôle de tiers doivent être pris en compte s'ils sont physical properties under third-party control should be considered if
pertinents. relevant.
La réponse (B) est juste. Le champ de la mission doit couvrir les Answer (B) is correct. The scope of the engagement should
systèmes, les documents, le personnel et les biens concernés, y include consideration of relevant systems, records,
compris ceux qui se trouvent sous le contrôle de tiers (CRIPP). Les personnel, and physical properties, including those under the
résultats de la mission rapportés dans les communications finales de control of third parties (IPPF). Engagement results reported in
la mission émanent de l'exécution des procédures et de la réalisation final engagement communications follow from performing the
des objectifs qui définissent le champ de la mission. procedures and achieving the objectives that define the
La réponse (C) est fausse. Les résultats de la mission rapportés dans les engagement scope.
communications finales de la mission émanent de l'exécution des procédures Answer (C) is incorrect because engagement results reported in
et de la réalisation des objectifs qui définissent le champ de la mission. final engagement communications follow from performing the
La réponse (D) est fausse. Les résultats de la mission rapportés dans les procedures and achieving the objectives that define the
communications finales de la mission émanent de l'exécution des procédures engagement scope.
et de la réalisation des objectifs qui définissent le champ de la mission. Answer (D) is incorrect because engagement results reported in
final engagement communications follow from performing the
procedures and achieving the objectives that define the
engagement scope.
Restitution
La réponse (A) est juste. Le programme de la mission doit documenter Answer (A) is correct. Work programs document procedures
les procédures permettant à l'auditeur interne de collecter, analyser, for collecting, analyzing, interpreting, and documenting
interpréter et documenter les informations pendant la mission ; définir information; state engagement objectives; set forth the scope
les objectifs de la mission ; établir le champ d'intervention et le degré and degree of testing needed to achieve objectives in each
d'approfondissement nécessaire des tests pour atteindre les objectifs phase of the engagement; identify technical aspects, risks,
de chaque phase de la mission ; identifier les aspects techniques, les processes, and transactions to be examined; state the nature
risques, les procédés et les transactions qui doivent être audités ; and extent of testing required, and are prepared before work
établir la nature et l'étendue nécessaires des tests ; et être préparé begins, with appropriate modification during the engagement
avant le démarrage de la mission, et modifié, si nécessaire, au cours de (IPPF). However, a work program must be adapted to the
la mission (CRIPP). Cependant, le programme de la mission doit être specific needs of the engagement after the internal auditor
adapté aux besoins spécifiques de cette dernière une fois que establishes the engagement objectives and scope and
l'auditeur interne a fixé les objectifs et le champ de la mission et determines the resources required.
déterminé les ressources requises. Answer (B) is incorrect because a generalized program not allows
La réponse (B) est fausse. Un programme standard ne tient pas compte des for variations resulting from changing circumstances and varied
variations résultant de l'évolution des circonstances et de l'environnement. conditions.
La réponse (C) est fausse. Un programme standard ne peut tenir compte des Answer (C) is incorrect because a generalized program cannot
variations des circonstances et des environnements différents. consider variations in circumstances and conditions.
La réponse (D) est fausse, car tous les aspects d'une opération n'ont pas Answer (D) is incorrect because every aspect of an operation need
besoin d'être examinés, mais uniquement ceux susceptibles de dissimuler not be examined, only those aspects likely to conceal problems
des problèmes et des difficultés. and difficulties.
P1 - 90 -- Inclure un budget temps dans le programme de travail d'une P1 - 90 -- The purpose of including a time budget in an
mission vise à : engagement work program is to
A. Procurer un moyen objectif d'évaluation de la compétence de A. Provide an objective means of evaluating the internal auditor's
l'auditeur interne. competence.
B. Garantir la réalisation de la mission dans les délais. B. Assure timely completion of the engagement.
C. Se doter d'un moyen de contrôle et d'évaluation de l'avancement de la mission. C. Provide a means of controlling and evaluating the progress of
D. Limiter le champ de la mission. the engagement.
D. Restrict the scope of the engagement.
La réponse (A) est fausse. De nombreux facteurs autres que la compétence Answer (A) is incorrect because whether an internal auditor
professionnelle influent sur la capacité d'un auditeur interne à ne pas remains within the time budget is affected by many factors other
dépasser le budget temps. than professional competence.
La réponse (B) est fausse. L'établissement d'un budget ne garantit pas la Answer (B) is incorrect because the establishment of a budget
réalisation du travail dans les délais. cannot assure that work will be completed on a timely basis.
La réponse (C) est juste. Les auditeurs internes doivent élaborer un Answer (C) is correct. Internal auditors should develop and
programme de travail permettant d'atteindre les objectifs de la mission record work programs to achieve the engagement objectives
(CRIPP). Ce programme doit répertorier les procédures à suivre (IPPF). The work program lists the procedures to be followed
pendant la mission. Il constitue donc un outil utile pour la during the engagement. Accordingly, a work program is a
programmation et le contrôle (supervision) de la mission. La useful tool for scheduling and controlling (supervising) the
supervision consiste notamment à vérifier que le programme approuvé engagement. Supervision includes, among other things,
est correctement exécuté sous réserve des modifications justifiées et determining that the approved work program is carried out
autorisées. De plus, la supervision inclut le suivi des dépenses et du unless changes are justified and authorized. Moreover,
temps passé (CRIPP). À cette fin, un budget temps est nécessaire pour supervision extends to time and expense control (IPPF). For
l'évaluation et le contrôle de l'avancement de la mission. Il permet de this purpose, a time budget is necessary to evaluate and
comparer le temps réel passé sur une procédure au temps alloué. control the progress of the engagement. It permits
La réponse (D) est fausse. Un budget temps n'a pas pour finalité de limiter le comparison of the actual time spent on a procedure with its
champ de la mission. allotted time.
Answer (D) is incorrect because a time budget is not intended to
limit the scope of the engagement.
Restitution
Restitution
La réponse (A) est fausse. L'auditeur interne doit essayer de Answer (A) is incorrect because the internal auditor should attempt
déterminer les causes des observations de la mission et, si to determine the causes of engagement observations and, if
nécessaire, les inclure dans les rapports. appropriate, include them in the engagement communication.
La réponse (B) est juste. Les auditeurs internes doivent communiquer Answer (B) is correct. Internal auditors should report the
les résultats de la missions rapidement (CRIPP). De plus, “le engagement results promptly (IPPF). Moreover, the CAE
responsable de l'audit interne doit mettre en place un processus de should establish a follow-up process to monitor and ensure
suivi permettant de surveiller et de garantir que des mesures ont été that management actions have been effectively implemented
effectivement mises en œuvre par le management ou que la Direction or that senior management has accepted the risk of not taking
Générale a accepté de prendre le risque de ne rien faire” (CRIPP). En action (IPPF). Furthermore, failure to disclose known material
outre, la non communication des faits matériels dont l'auditeur interne facts that, if not disclosed, may distort the reporting of
a connaissance et qui, s'ils n'étaient pas révélés, auraient pour activities under review is a violation of Conduct Rule 2.3.
conséquence de fausser le rapport sur les activités examinées Answer (C) is incorrect because the engagement client's view about
constitue une violation de la Règle de Conduite 2.3. engagement observations, conclusions, and recommendations
La réponse (C) est fausse. L'opinion du client de la mission sur les should be included in the engagement communication.
observations, conclusions et recommandations de la mission doit figurer dans Answer (D) is incorrect because failure to report the deviations may
les communications. be justified if the internal auditor has concluded that the amounts are
La réponse (D) est fausse. La non communication des écarts peut se justifier clearly not material, that they are not in violation of governmental
si l'auditeur interne juge que leurs montants ne sont pas significatifs, qu'ils regulations, and that a rationale for the deviations exists.
n'enfreignent pas la loi et qu'ils peuvent s'expliquer.
Restitution
P1 - 94 -- Un auditeur interne vient d'être informé de la prochaine mission, et P1 - 94 -- The internal auditor-in-charge has just been informed of
l'équipe a été constituée. Sélectionner la phase appropriée pour finaliser le the next engagement, and the engagement team has been
budget de la mission : assigned. Select the appropriate phase for finalizing the
engagement budget.
A. Pendant la formulation du plan à long terme.
B. Après l'examen préliminaire. A. During formulation of the long-range plan.
C. Durant la réunion de planification initiale. B. After the preliminary survey.
D. Après l'achèvement de tous les travaux sur place. C. During the initial planning meeting.
D. After the completion of all field work.
La réponse (A) est fausse. Un budget initial est fixé pendant la formulation du Answer (A) is incorrect because an initial budget is determined
plan à long terme, mais des révisions sur la base de l'examen préliminaire during the formulation of the long-range plan, but revisions based
peuvent se révéler nécessaires. on the preliminary survey may be required.
La réponse (B) est juste. “Un examen préliminaire permet une Answer (B) is correct. A survey permits an informed approach
approche documentée pour planifier et réaliser la mission. C'est un to planning and carrying out engagement work and is an
moyen efficace pour affecter les ressources de l'audit interne aux effective tool for applying the IAA's resources where they can
secteurs où elles peuvent être utilisées le plus efficacement”. Ses be used most effectively. Among other things, the results of
résultats doivent entre autres comprendre “les premières estimations the survey should include preliminary estimates of time and
en matière de délai et de besoin en ressources” (CRIPP). Ainsi, une fois resource requirements (IPPF). Thus, after the preliminary
l'examen préliminaire achevé, le budget final de la mission peut être survey has been completed, the final engagement budget can
élaboré. be prepared.
La réponse (C) est fausse. Lors de la réunion de planification initiale, le projet Answer (C) is incorrect because at the initial planning meeting
n'est pas suffisamment défini pour déterminer le budget final. stage, the project is not sufficiently defined to complete the final
La réponse (D) est fausse. Après l'achèvement des travaux sur place, le budget.
budget ne sert plus d'outil de contrôle et d'évaluation. Answer (D) is incorrect because, after the completion of the field
work, the budget is no longer useful as a control and evaluation
tool.
Restitution
Restitution
La réponse (A) est juste. La meilleure méthode consiste à sélectionner Answer (A) is correct. The best method is to take an attribute
par sondage pour estimation d'attributs un échantillon des sample from the log of customer complaints. Whether
réclamations des clients. Elle permet d'obtenir uniquement des inquiries have been satisfied promptly is a yes/no, either/or
réponses de type oui/non et ou/ou à la question portant sur le délai de proposition appropriate for this sampling method. The
traitement des réclamations. Les éléments échantillonnés doivent être sampled items should be traced to documentation that
comparés aux documents qui doivent faire apparaître l'intervention du should evidence the completion of customer service.
service après-vente. Answer (B) is incorrect because surveying all customers would be
La réponse (B) est fausse. Il serait onéreux de questionner tous les clients. costly. The procedure also has two additional faults: (1) the
Cette procédure présente deux autres inconvénients : (1) le client ne se customer might not recall the time necessary for the repair, and (2)
souvient pas toujours du délai d'intervention qui a été nécessaire et (2) il ne the customer might not respond.
répondra pas forcément. Answer (C) is incorrect because surveying a sample of customers
La réponse (C) est fausse. La méthode consistant à questionner un suffers from the same two faults noted above. Moreover, MPU is a
échantillon de clients présente les deux inconvénients décrits ci-dessus. De variables sampling technique that is appropriate for testing values,
plus, le sondage pour estimation de variables par la moyenne est une not binary propositions.
technique de sondage pour estimation de variables qui est appropriée pour Answer (D) is incorrect because sampling from the total customer
vérifier des valeurs, mais pas des propositions binaires. population is inefficient. Not all customers have complained during
La réponse (D) est fausse. La méthode consistant à prendre un échantillon the year. Also MPU is inappropriate.
de la population totale de clients est inefficiente. En effet, les clients ne se
sont pas tous plaints au cours de l'année et le sondage pour estimation de
variables par la moyenne est inapproprié.
Restitution
Restitution
Restitution
P1 - 99 -- Vous êtes le responsable de l'audit interne d'une grande P1 - 99 -- You are the chief audit executive for a large decentralized
organisation décentralisée. Vous avez élaboré un manuel qui contient des organization. You have developed a manual containing
procédures écrites détaillées et complètes pour guider les groupes de travail comprehensive detailed written procedures as a guide for your
décentralisés. Chacun de ces groupes compte 20 à 30 auditeurs internes. decentralized engagement work groups, each of which has 20 to
L'organisation vient d'acquérir une petite entité qui possède un petit service 30 internal auditors. The organization recently acquired a small
d'audit interne composé d'un superviseur et de deux auditeurs. Laquelle des entity that has an internal audit activity consisting of a supervisor
actions suivantes sera la mieux à même de donner une orientation and two staff personnel. Which of the following actions is the most
administrative au nouveau service d'audit interne ? practical in providing administrative guidance for this new internal
audit activity?
A. Sélectionner des procédures clés dans le manuel et utiliser les indications A. Select key procedures from the manual and use informal
informelles des superviseurs pour les autres questions relatives au supervisory direction for other engagement management issues.
management de la mission. B. Use informal supervisory direction for engagement management
B. Utiliser les indications informelles des superviseurs pour les questions issues.
relatives au management de la mission. C. Use the already developed manual.
C. Utilisé le manuel déjà élaboré. D. Adopt the administrative procedures being followed by the
D. Adopter les procédures administratives suivies par les auditeurs internes internal auditors of the acquired entity.
de l'entité rachetée.
La réponse (A) est juste. Afin de familiariser le personnel de l'entité Answer (A) is correct. Orientation to acquaint the acquired
nouvellement acquise avec son nouvel environnement, il convient de entity's staff with the established environment should be
l'exposer à des procédures clés sélectionnées et extraites du manuel. through exposure to selected key procedures from the formal
La forme et le contenu des règles et procédures écrites doivent manual. The form and content of written policies and
correspondre à la taille et à la structure du service d'audit interne. Un procedures should be appropriate to the size and structure of
petit service d'audit interne peut être dirigé de manière moins formelle. the IAA. Thus, a small IAA may be managed less formally, for
Il peut par exemple être “contrôlé au jour le jour par une supervision example, through daily close supervision and written
étroite et des notes de service” (CRIPP). memoranda (IPPF).
La réponse (B) est fausse. L'utilisation de la seule supervision informelle pour Answer (B) is incorrect because the use of informal supervisory
du personnel nouvellement embauché est inadéquate. direction alone for new staff is inadequate.
La réponse (C) est fausse. L'utilisation du manuel existant dans son Answer (C) is incorrect because complete reliance on the existing
intégralité nécessiterait de diriger le service de manière plus formelle que manual would require more formal management than is necessary
nécessaire pour un petit service d'audit interne. for a small IAA.
La réponse (D) est fausse. Le management de la nouvelle entité d'audit Answer (D) is incorrect because management of the new internal
interne doit être en droite ligne de celui du reste de l'organisation. auditing entity should not be inconsistent with the rest of the
organization.
Restitution
P1 - 101 -- Le développement organisationnel (DO) est l'une des grandes P1 - 101 -- Organizational development (OD) is one of the major
approches de la gestion dynamique du changement dans les organisations. approaches to proactive management of change in organizations.
L'un de ses principaux objectifs est : One of the major objectives of OD is to
A. De renforcer le pouvoir des leaders. A. Increase the power of leaders.
B. D'aligner les objectifs de l'organisation et des salariés. B. Align the organization's and the employees' goals.
C. D'attirer de meilleurs éléments au sein de l'organisation. C. Attract better employees to the organization.
D. De fournir à l'organisation et à ses dirigeants des moyens d'accroître D. Provide the organization and its managers with ways to increase
l'efficience. efficiency.
La réponse (A) est fausse. Le DO est axé sur la participation et le partage Answer (A) is incorrect because OD focuses on participation and
des pouvoirs. power sharing.
La réponse (B) est juste. Le DO a pour but d'approfondir le sens de Answer (B) is correct. The objectives of OD are to deepen the
l'objectif organisationnel et de faire en sorte que les individus y sense of organizational purpose and align individuals with it;
adhèrent, de favoriser la confiance, la communication, la coopération et to promote interpersonal trust, communication, cooperation,
le soutien entre les individus, de promouvoir une approche axée sur la and support; to encourage a problem-solving approach; to
résolution des problèmes, de rendre l'expérience professionnelle develop a satisfying work experience; to supplement formal
satisfaisante, de compléter l'autorité formelle par une autorité fondée authority with authority based on expertise; to increase
sur le savoir-faire, de conférer davantage de poids à la responsabilité personal responsibility; and to encourage willingness to
personnelle et d'encourager la volonté de changement. change.
La réponse (C) est fausse. Attirer de meilleurs éléments au sein d'une Answer (C) is incorrect because attracting better applicants to an
organisation ne fait pas partie des objectifs premiers du DO. organization is not a major goal of OD.
La réponse (D) est fausse. Le DO accroît l'efficacité pour l'organisation et sa Answer (D) is incorrect because OD provides an organization and
direction. its managers with higher effectiveness.
Restitution
P1 - 103 -- La fonction de directeur du risque (chief risk officer - CRO) est P1 - 103 -- The function of the chief risk officer (CRO) is most
surtout efficace lorsque celui-ci : effective when the CRO:
A. Gère le risque en sa qualité de membre de la direction. A. Manages risk as a member of senior management.
B. Partage la gestion du risque avec l'encadrement intermédiaire. B. Shares the management of risk with line management.
C. Partage la gestion du risque avec le responsable de l'audit interne. C. Shares the management of risk with the chief audit executive.
D. Supervise le risque au sein de l'équipe de gestion du risque de l'entreprise. D. Monitors risk as part of the enterprise risk management team.
La réponse (A) est fausse. L'encadrement supérieur a une mission de Answer (A) is incorrect. Senior management has an oversight role
supervision dans le cadre de la gestion du risque. in risk management.
La réponse (B) est fausse. Le niveau de l'encadrement intermédiaire a une Answer (B) is incorrect. The risk knowledge at the line level would
connaissance du risque cantonnée à sa sphère d'activité. be specific only to that area of the organization.
La réponse (C) est fausse. Le responsable de l'audit interne n'est pas chargé Answer (C) is incorrect. The chief audit executive (CAE) does not
de la gestion du risque. have the responsibility for managing risk.
La réponse (D) est juste. Le directeur du risque est le plus efficace Answer (D) is correct. The chief risk officer is most effective
lorsqu'il est épaulé par une équipe disposant du savoir-faire et de when supported by a specific team with the necessary
l'expérience requis concernant les risques supportés par expertise and experience related to organizational risk.
l'organisation.
Restitution
P1 - 105 -- Laquelle des affirmations suivantes sur les systèmes de P1 - 105 -- Which of the following statements is correct regarding
rémunération et de prime d'une entreprise est-elle correcte ? corporate compensation systems and related bonuses?
I. Un système de prime doit être considéré comme faisant partie de I. A bonus system should be considered part of the control
l'environnement de contrôle d'une organisation et doit être pris en compte environment of an organization and should be considered in
dans la formulation d'un rapport sur le contrôle interne. formulating a report on internal control.
II. Les systèmes de rémunération ne font pas partie de l'environnement de II. Compensation systems are not part of an organization's control
contrôle d'une organisation et les rapports ne doivent donc pas en tenir system and should not be reported as such.
compte. III. An audit of an organization's compensation system should be
III. Il convient d'effectuer l'audit du système de rémunération d'une entreprise performed independently of an audit of the control system over
indépendamment de celui du système de contrôle sur les autres fonctions other functions that impact corporate bonuses.
qui exercent une influence sur les primes.
A. I only.
A. I uniquement. B. II only.
B. II uniquement. C. III only.
C. III uniquement. D. II and III only.
D. II et III uniquement.
La réponse (A) est juste. Les systèmes de rémunération influencent le Answer (A) is correct. Compensation systems influence
comportement et doivent donc être considérés comme faisant partie behavior and should be considered an integral part of an
intégrante de la structure de contrôle d'une organisation. organization's control structure.
La réponse (B) est fausse. Les systèmes de rémunération font partie des Answer (B) is incorrect. Compensation systems are part of the
systèmes de contrôle de l'organisation. organization's control systems.
La réponse (C) est fausse. Les audits des systèmes de rémunération Answer (C) is incorrect. Audits of the compensation systems can
peuvent être associés à un audit d'autres fonctions qui influent sur les primes. be combined with an audit over other functions that impact
La réponse (D) est fausse. Les audits des systèmes de rémunération corporate bonuses.
peuvent être associés à un audit d'autres fonctions qui influent sur les primes. Answer (D) is incorrect. Compensation systems are part of the
organization's control systems. Audits of the compensation
systems can be combined with an audit over other functions that
impact corporate bonuses.
Restitution
Restitution
P1 - 109 -- Lequel des objectifs suivants optimise les stratégies de gestion du P1 - 109 -- Which of the following goals sets risk management
risque ? strategies at the optimum level?
A. Minimiser les coûts. A. Minimize costs.
B. Maximiser la part de marché. B. Maximize market share.
C. Minimiser les pertes. C. Minimize losses.
D. Maximiser la valeur pour les actionnaires. D. Maximize shareholder value.
La réponse (A) est fausse. Ce n'est pas une approche complète de la gestion Answer (A) is incorrect. This is not a comprehensive approach to
du risque. risk management.
La réponse (B) est fausse. Voir réponse A. Answer (B) is incorrect. See answer "a".
La réponse (C) est fausse. Voir réponse A. Answer (C) is incorrect. See answer "a".
La réponse (D) est juste. C'est une approche complète, en liaison avec Answer (D) is correct. This is a comprehensive approach and
les stratégies de gestion du risque dans toute l'entreprise. will relate to risk management strategies across the
enterprise.
Restitution
P1 - 111 -- Tous les éléments suivants font partie du système de contrôle P1 - 111 -- All of the following would be part of a factory's control
d'une usine destiné à éviter les rejets d'effluents liquides non conformes aux system to prevent release of waste water that does not meet
normes sauf : discharge standards except:
A. Procéder à des analyses chimiques de l'eau avant le rejet pour détecter A. Performing chemical analysis of the water, prior to discharge, for
les composants énumérés dans l'autorisation. components specified in the permit.
B. Spécifier (dans la politique d'entreprise, par une formation ou par B. Specifying (by policy, training, and advisory signs) which
l'affichage de recommandations) les substances qu'il est permis d'évacuer substances may be disposed of via sinks and floor drains within the
par un évier ou les orifices d'évacuation dans le sol dans l'enceinte de l'usine. factory.
C. Laver périodiquement les éviers et les orifices d'évacuation dans le sol à C. Periodically flushing sinks and floor drains with a large volume of
grande eau afin que les substances polluantes soient suffisamment diluées. clean water to ensure pollutants are sufficiently diluted.
D. Mettre en place un programme de maintenance préventive pour le D. Establishing a preventive maintenance program for the factory's
système de prétraitement de l'usine. pretreatment system.
La réponse (A) est fausse. (comme B et D). Pris individuellement, chacun de Answer (A) is incorrect (like B and D). Each of these individual
ces contrôles, ainsi que d'autres probablement, aide la direction à parvenir à controls, and probably others as well, help management achieve its
son objectif, à savoir éviter le rejet d'effluents liquides non conformes aux objective of preventing the release of waste water that does not
plafonds autorisés ou à d'autres normes. Ces trois contrôles ont chacun une meet permit limits or other conditions. These three controls each
approche différente du risque. Les résultats des analyses constituent les approach the risk in different ways. Analytical results are the criteria
critères de décision sur les rejets ; le fait de ne pas évacuer les substances for the decision to discharge; keeping pollutants out of the waste
polluantes dans les eaux usées permet de réduire les concentrations et le water will help reduce concentrations and the degree of
degré de prétraitement nécessaire, et les pannes de matériel sont moins pretreatment needed; and equipment breakdown is less likely to
probables si un programme de maintenance préventive est en place. occur if a preventive maintenance program is in place.
La réponse (B) est fausse. (comme A et D). Pris individuellement, chacun de Answer (B) is incorrect (like A and D). Each of these individual
ces contrôles, ainsi que d'autres probablement, aide la direction à parvenir à controls, and probably others as well, help management achieve its
son objectif, à savoir éviter le rejet d'effluents liquides non conformes aux objective of preventing the release of waste water that does not
plafonds autorisés ou à d'autres normes. Ces trois contrôles ont chacun une meet permit limits or other conditions. These three controls each
approche différente du risque. Les résultats des analyses constituent les approach the risk in different ways. Analytical results are the criteria
critères de décision sur les rejets ; le fait de ne pas évacuer les substances for the decision to discharge; keeping pollutants out of the waste
polluantes dans les eaux usées permet de réduire les concentrations et le water will help reduce concentrations and the degree of
degré de prétraitement nécessaire, et les pannes de matériel sont moins pretreatment needed; and equipment breakdown is less likely to
probables si un programme de maintenance préventive est en place. occur if a preventive maintenance program is in place.
La réponse (C) est juste. Une dilution périodique ne peut pas toujours Answer (C) is correct. Periodic dilution may not always
prévenir les rejets de substances polluantes qui dépassent les limites prevent the release of pollutants which exceed the discharge
autorisées. limits.
La réponse (D) est fausse. (comme A et B). Pris individuellement, chacun de Answer (D) is incorrect (like A and B). Each of these individual
ces contrôles, ainsi que d'autres probablement, aide la direction à parvenir à controls, and probably others as well, help management achieve its
son objectif, à savoir éviter le rejet d'effluents liquides non conformes aux objective of preventing the release of waste water that does not
plafonds autorisés ou à d'autres normes. Ces trois contrôles ont chacun une meet permit limits or other conditions. These three controls each
approche différente du risque. Les résultats des analyses constituent les approach the risk in different ways. Analytical results are the criteria
critères de décision sur les rejets ; le fait de ne pas évacuer les substances for the decision to discharge; keeping pollutants out of the waste
polluantes dans les eaux usées permet de réduire les concentrations et le water will help reduce concentrations and the degree of
degré de prétraitement nécessaire, et les pannes de matériel sont moins pretreatment needed; and equipment breakdown is less likely to
probables si un programme de maintenance préventive est en place. occur if a preventive maintenance program is in place.
Restitution
P1 - 113 -- Le changement organisationnel doit être considéré en tenant P1 - 113 -- Organizational change must be considered in the light
compte de la résistance que peut lui opposer le personnel. Cette résistance : of potential employee resistance. Resistance
A. Peut survenir alors même que le personnel bénéficiera de ce changement. A. May occur even though employees will benefit from the change.
B. Sera la plus vive lorsque les groupes informels sont les plus faibles. B. Will be greatest when informal groups are weakest.
C. Sera insignifiante si le personnel ne redoute pas de pertes financières. C. Will be insignificant if no economic loss by employees is
D. Est essentiellement axée sur les menaces perçues sur les besoins expected.
psychologiques. D. Is centered mostly on perceived threats to psychological needs.
La réponse (A) est juste. La résistance au changement peut être Answer (A) is correct. Resistance to change may be caused
provoquée par la crainte des ajustements personnels qui peuvent se by fear of the personal adjustments that may be required.
révéler nécessaires. Le personnel peut être véritablement inquiet de la Employees may have a genuine concern about the
pertinence du changement, percevoir un manque d'égard envers ses usefulness of the change, perceive a lack of concern for
sentiments, craindre le résultat, s'inquiéter d'une dévalorisation de son workers' feelings, fear the outcome, worry about downgrading
statut professionnel et redouter l'abandon des anciennes procédures of job status, and resent deviations from past procedures for
(en particulier si les nouvelles sont moins participatives que les implementing change (especially if new procedures are less
anciennes). Des ajustements sociaux peuvent également se révéler participative than the old). Social adjustments also may be
nécessaires, qui enfreignent les normes comportementales des required that violate the behavioral norms of informal groups
groupes informels ou dérangent le statut quo social au sein des or disrupt the social status quo within groups. Economic
groupes. Les ajustements économiques peuvent entraîner des pertes adjustments may involve potential economic loss or
économiques ou une insécurité due aux menaces perçues envers les insecurity based on perceived threats to jobs. In general, any
emplois. En général, toute dégradation perçue de la situation perceived deterioration in the work situation that is seen as a
professionnelle qui est considérée comme une menace sur les besoins threat to economic, social, and/or psychological needs will
économiques, sociaux et/ou psychologiques produira de la résistance. produce resistance. The various adjustments required are
Les divers ajustements nécessaires risquent davantage de se heurter à most likely to be resisted when imposed unilaterally by higher
une résistance lorsqu'ils sont imposés de manière unilatérale par une authority. However, employees who share in finding solutions
autorité supérieure. Cependant, les salariés qui participent à la to the problems requiring change are less likely to resist
recherche de solutions aux problèmes nécessitant un changement because they will have some responsibility for the change.
sont moins susceptibles de s'opposer car ils ont une certaine Answer (B) is incorrect because strong informal groups are likely to
responsabilité dans le changement. offer more resistance.
La réponse (B) est fausse, car si les groupes informels sont puissants, ils Answer (C) is incorrect because resistance arises from threats to a
risquent d'opposer davantage de résistance. complex pattern of economic, social, and psychological needs.
La réponse (C) est fausse. La résistance provient des menaces qui pèsent Answer (D) is incorrect because resistance arises from threats to a
sur un schéma complexe de besoins économiques, sociaux et complex pattern of economic, social, and psychological needs.
psychologiques.
La réponse (D) est fausse. La résistance provient des menaces qui pèsent
sur un schéma complexe de besoins économiques, sociaux et
psychologiques.
Restitution
P1 - 115 -- Laquelle des observations suivantes indiquera le plus P1 - 115 -- Which of the following observations by an auditor is
probablement à l'auditeur l'existence de carences dans les contrôles sur la most likely to indicate the existence of control weaknesses over
préservation des actifs ? safeguarding of assets?
I. En raison de son emplacement, un service ne peut pas servir I. A service department's location is not well suited to allow
correctement les autres unités. adequate service to other units.
II. Les antécédents des salariés occupant des postes sensibles ne sont pas II. Employees hired for sensitive positions are not subjected to
contrôlés. background checks.
III. Les cadres n'ont pas accès aux rapports présentant les performances III. Managers do not have access to reports that profile overall
globales de l'entreprise par rapport à celles des autres organisations de performance in relation to other benchmarked organizations.
référence. IV. Management has not taken corrective action to resolve past
IV. La direction n'a pas pris de mesures correctives pour remédier aux engagement observations related to inventory controls.
lacunes observées lors de missions précédentes dans les contrôles des
stocks. A. I and II only.
B. I and IV only.
A. I et II uniquement. C. II and III only.
B. I et IV uniquement. D. II and IV only.
C. II et III uniquement.
D. II et IV uniquement.
La réponse (A) est fausse. La proposition I décrit un symptôme de carences Answer (A) is incorrect. The item I is a symptom of weak controls
dans les contrôles mis en place pour la réalisation des buts et objectifs de for achieving organizational goals and objectives, but not for
l'organisation, mais non pour la préservation des actifs. safeguarding of assets.
La réponse (B) est fausse. La proposition I décrit un symptôme de carences Answer (B) is incorrect. The item I is a symptom of weak controls
dans les contrôles mis en place pour la réalisation des buts et objectifs de for achieving organizational goals and objectives, but not for
l'organisation, mais non pour la préservation des actifs. safeguarding of assets.
La réponse (C) est fausse. La proposition III décrit un symptôme de carences Answer (C) is incorrect. The item III is a symptom of weak controls
dans les contrôles mis en place pour la réalisation des buts et objectifs de for achieving organizational goals and objectives, but not for
l'organisation, mais non pour la préservation des actifs. safeguarding of assets.
La réponse (D) est juste. La proposition II décrit un symptôme de Answer (D) is correct. The item II is a symptom of weak
carences dans les contrôles mis en place pour la préservation des controls for achieving organizational goals and objectives,
actifs.Le fait que la direction n'ait pas pris de mesures correctives à la but not for safeguarding of assets. Management's failure to
suite des observations des missions précédentes, qui portaient sur la take corrective action on past engagement observations,
préservation des actifs, constitue une carence en la matière. which related to safeguarding of assets, is a weakness related
to safeguarding of assets.
Restitution
P1 - 117 -- La filiale d'une multinationale comporte une unité de virements de P1 - 117 -- Appropriate internal control for a multinational
fonds. Lequel des contrôles internes suivants est impératif ? corporation's branch office that has a monetary transfer unit
requires that:
A. La personne qui procède aux virements ne fait pas les rapprochements A. The individual who initiates wire transfers not reconcile the bank
bancaires. statement.
B. C'est le responsable de la filiale qui reçoit l'ensemble des virements. B. The branch manager receive all wire transfers.
C. Les taux de change doivent être calculés séparément par deux employés C. Foreign currency rates be computed separately by two different
différents. employees.
D. Ce sont les cadres de l'entreprise qui avalisent l'embauche des employés D. Corporate management approve the hiring of monetary transfer
de l'unité des virements. unit employees.
La réponse (A) est juste. Il est nécessaire que le rapprochement Answer (A) is correct. Independent reconciliation of bank
bancaire soit indépendant pour que le contrôle interne soit satisfaisant. accounts is necessary for good internal control.
La réponse (B) est fausse. Cette considération n'est pas importante pour le Answer (B) is incorrect. This is not an important internal control
contrôle interne. consideration.
La réponse (C) est fausse. La conversion au moyen des taux de change Answer (C) is incorrect. Foreign currency translation rates are not
n'est pas calculée, mais vérifiée. Le fait que deux employés du même computed, but instead verified. Having two employees in the same
département effectuent la même tâche n'améliorera pas significativement le department perform the same task will not significantly enhance
contrôle interne. internal control.
La réponse (D) est fausse. Cette considération n'est pas importante pour le Answer (D) is incorrect. This is not an important internal control
contrôle interne. consideration.
P1 - 118 -- Laquelle des propositions suivantes ne fait pas partie de la P1 - 118 -- Which of the following is not implied by the definition of
définition du contrôle ? control?
A. Mesurer les avancées en direction des objectifs. A. Measurement of progress toward goals.
B. Révéler les écarts par rapports aux plans. B. Uncovering of deviations from plans.
C. Attribuer les responsabilités concernant les écarts. C. Assignment of responsibility for deviations.
D. Indiquer la nécessité d'une mesure correctrice. D. Indication of the need for corrective action.
La réponse (A) est fausse. La mesure des avancées en direction des buts est Answer (A) is incorrect because measurement of progress toward
sous-entendue dans la définition du contrôle. goals is implied by the definition of control.
La réponse (B) est fausse. L'identification des écarts par rapport aux plans Answer (B) is incorrect because uncovering of deviations from
est sous-entendue dans la définition du contrôle. plans is implied by the definition of control.
La réponse (C) est juste. Pour l'encadrement, le contrôle consiste Answer (C) is correct. The basic process of control for
fondamentalement à définir des normes, mesurer les performances à managers consists of establishing standards, measuring
l'aune de ces références et corriger les écarts afin de garantir la performance against standards, and correcting for deviations
réalisation des objectifs de l'entreprise. Ainsi, l'attribution des to ensure accomplishment of enterprise goals. Thus,
responsabilités concernant les écarts observés ne fait pas partie du assigning responsibility for deviations found is not a part of
contrôle. the controlling function.
La réponse (D) est fausse. Le fait d'indiquer la nécessité d'une mesure Answer (D) is incorrect because indication of the need for
correctrice est sous-entendu dans la définition du contrôle. corrective action is implied by the definition of control.
Restitution
P1 - 120 -- Lorsqu'on détermine l'éventail de contrôle approprié, il faut avant P1 - 120 -- When determining the appropriate span of control, the
tout prendre en compte : most important consideration is
A. L'ensemble des politiques et procédures en vigueur. A. The set of policies and procedures currently in effect.
B. L'éventail de contrôle type dans les autres organisations. B. The typical span of control used by other organizations.
C. Les préférences des créanciers de l'organisation. C. The preference of the organization's creditors.
D. Le fait que tous les services seront évalués, qu'ils soient ou non D. That all departments will be evaluated, whether or not they will
concernés. be affected.
La réponse (A) est juste. Les facteurs les plus importants à prendre à Answer (A) is correct. The most important factors to consider
compte sont les préférences et qualifications du personnel et de are the employees' and manager's preferences and skills, the
l'encadrement, la culture de l'organisation, les tâches concernées, organization's culture, the tasks involved, physical location of
l'emplacement du service, ainsi que les politiques et procédures the department, and established policies and procedures.
établies. Answer (B) is incorrect because, although examining similar
La réponse (B) est fausse, car même s'il peut être utile d'examiner ce que entities may be useful, it is not one of the most important
font des entités analogues, ce n'est pas le facteur le plus important à prendre considerations.
en compte. Answer (C) is incorrect because this consideration would only be
La réponse (C) est fausse. La prise en compte de ce facteur n'est nécessaire necessary if required by an agreement with the creditors.
que si un accord passé avec les créanciers l'impose. Answer (D) is incorrect because whether all departments will be
La réponse (D) est fausse. Une décision portant sur l'éventail de contrôle ne evaluated is not normally considered in a span-of-control decision.
tient habituellement pas compte de la question de savoir si tous les services
seront évalués.
P1 - 121 -- Pendant une mission d’audit, l’auditeur a éprouvé une certaine P1 - 121 -- During an internal audit, the auditor experienced
difficulté pour obtenir les informations dont il avait besoin auprès d’un salarié. difficulty obtaining required information from a specific employee.
Comme cette situation a perduré pendant une semaine, l’auditeur a When this situation continued for one week, the auditor requested
demandé un rendez-vous avec le salarié afin d’identifier le problème et de a private meeting with the employee for the purpose of identifying
résoudre la difficulté grâce à une discussion ouverte. Quelle technique de the problem and resolving the difficulty through open discussion.
gestion de conflit utilisait l’auditeur ? Which conflict management technique was the auditor applying?
A. La résolution des problèmes. A. Problem solving.
B. L’élargissement des ressources B. Expansion of resources.
C. La commande autoritaire C. Authoritative command.
D. La modification de la variable humaine. D. Altering the human variable.
La réponse (A) est juste. La technique de gestion de conflit qui implique Answer (A) is correct. The conflict management technique
l'organisation de réunions face à face est la résolution des problèmes. that involves face-to-face meetings is problem solving.
La résolution des problèmes est une façon d’affronter le conflit et d’en Problem solving is a means of confronting the conflict and
faire disparaître les causes. L’accent est mis sur les faits et les removing its causes. The emphasis is on facts and solutions,
solutions, non pas sur les personnalités ou l'affectation de la not personalities and assignment of blame.
responsabilité. Answer (B) is incorrect because expansion of resources addresses
La réponse (B) est fausse. L’élargissement des ressources s’adresse aux conflicts that arise from scarcity.
conflits qui sont dus à la pénurie de ressources. Answer (C) is incorrect because the auditor is not using formal
La réponse (C) est fausse. L’auditeur n’utilise pas une autorité formelle. authority.
La réponse (D) est fausse. L’auditeur n’utilise pas des techniques Answer (D) is incorrect because the auditor is not using behavioral
comportementales afin de modifier des attitudes ou un comportement. techniques to change attitudes and behavior.
Restitution
Restitution
La réponse (A) est fausse. Des niveaux de dépense prédéfinis inclueraient Answer (A) is incorrect. Predefined spending levels would probably
probablement déjà les montants frauduleux et ne feraient que limiter already include the fraudulent amounts and would only limit the
l'ampleur de la fraude. size of the fraud.
La réponse (B) est juste. La solution qui permettrait le plus Answer (B) is correct. Additional authorization would be the
probablement d'empêcher cette fraude consiste à exiger une most likely choice in preventing the fraud.
autorisation complémentaire. Answer (C) is incorrect. The bill of lading would agree with the
La réponse (C) est fausse. Le connaissement serait conforme à la purchase order. The quantity received (verified by a third party)
commande. Il faudrait comparer la quantité reçue (vérifiée par un tiers) à la should be compared to both the bill of lading and the purchase
fois au connaissement et au bon de commande. order.
La réponse (D) est fausse. Le rapprochement informatique permettrait Answer (D) is incorrect. The computer matching would only verify
seulement de vérifier s'il y a eu des écritures frauduleuses. the fraudulent paperwork.
P1-125 -- Laquelle des entrées frauduleuses suivantes est la plus susceptible P1-125 -- Which of the following fraudulent entries is most likely to
d'être utilisée pour masquer le vol d'un actif ? be made to conceal the theft of an asset?
A. Débiter des charges et créditer l'actif. A. Debit expenses, and credit the asset.
B. Débiter l'actif et créditer un autre compte d'actif. B. Debit the asset, and credit another asset account.
C. Débiter les produits et créditer l'actif. C. Debit revenue, and credit the asset.
D. Débiter un autre compte d'actif et créditer l'actif. D. Debit another asset account, and credit the asset.
La réponse (A) est juste. La plupart des fraudeurs tenteront de Answer (A) is correct. Most fraud perpetrators would attempt
dissimuler leur vol en l'imputant sur un compte de charge. to conceal their theft by charging it against an expense
La réponse (B) est fausse. Débiter le compte correspondant à l'actif volé irait account.
dans la mauvaise direction si le but est de masquer un vol d'actif. Answer (B) is incorrect. Debiting the stolen asset account would be
La réponse (C) est fausse. Une entrée qui diminuerait les produits serait going in the wrong direction to conceal an asset theft.
inhabituelle et attirerait l'attention. Answer (C) is incorrect. An entry decreasing revenue would be
La réponse (D) est fausse. Cette entrée ne permettrait pas de masquer la unusual and would stand out.
fraude de manière permanente. Elle reporterait simplement un solde Answer (D) is incorrect. This entry would not permanently conceal
déséquilibré sur un autre compte. the fraud. It would simply shift the unreconcilible balance to another
asset account.
Restitution